Vision Geography

You might also like

Download as pdf or txt
Download as pdf or txt
You are on page 1of 48

Downloaded From:- https://telegram.me/pdf4exams https://telegram.

me/upsc_zone

VISION IAS
TEST SERIES

ENGLISH
Prelims Test
Test-12
Polity
GEOGRAPHY
https://www.facebook.com/pdf4exams/
Downloaded From:- https://telegram.me/pdf4exams https://telegram.me/upsc_zone

VISIONIAS
www.visionias.in

Test Booklet Series

TEST BOOKLET

GENERAL STUDIES (P) 2018 – Test–2439


C
Time Allowed: Two Hours Maximum Marks: 200

INSTRUCTIONS

1. IMMEDIATELY AFTER THE COMMENCEMENT OF THE EXAMINATION, YOU SHOULD CHECK THAT THIS BOOKLET
DOES NOT HAVE ANY UNPRINTED OR TURN OR MISSING PAGES OR ITEMS, ETC. IF SO, GET IT REPLACED BY A
COMPLETE TEST BOOKLET.

2. ENCODE CLEARLY THE TEST BOOKLET SERIES A, B, C OR D AS THE CASE MAY BE IN THE APPROPRIATE PLACE IN
THE ANSWER SHEET.

3. You have to enter your Roll Number on the Test Booklet in the Box
provided alongside. Do NOT write anything else on the Test Booklet.

4. This Test Booklet contains 100 items (Questions). Each item is printed in English. Each item comprises four
responses (answers). You will select the response which you want to mark on the Answer Sheet. In case you
feel that there is more than one correct response with you consider the best. In any case, choose ONLY ONE
response for each item.

5. You have to mark all your responses ONLY on the separate Answer Sheet provided. See direction in the
answers sheet.

6. All items carry equal marks. Attempt all items. Your total marks will depend only on the number of correct
responses marked by you in the answer sheet. For every incorrect response 1/3rdof the allotted marks will be
deducted.

7. Before you proceed to mark in the Answer sheet the response to various items in the Test booklet, you have to
fill in some particulars in the answer sheets as per instruction sent to you with your Admission Certificate.

8. After you have completed filling in all responses on the answer sheet and the examination has concluded, you
should hand over to Invigilator only the answer sheet. You are permitted to take away with you the Test
Booklet.

9. Sheet for rough work are appended in the Test Booklet at the end.

DO NOT OPEN THIS BOOKLET UNTIL YOU ARE ASKED TO DO SO


1 www.visionias.in ©Vision IAS

https://www.facebook.com/pdf4exams/
Downloaded From:- https://telegram.me/pdf4exams https://telegram.me/upsc_zone

1. Which of the following processes is 4. Which of the following statements regarding


responsible for generation of local winds in Geographical Indication (GI) is/are correct?
Northern India, known as 'loo'? 1. It is used for both agricultural and

(a) Conduction manufactured goods.

(b) Convection 2. The registration of a good as GI is valid

(c) Advection only for 10 years.

(d) Radiation 3. It is maintained under Ministry of


Commerce and Industry.

2. Which among the following include Select the correct answer using the code

economic activities of Mediterranean given below:

climate? (a) 1 only

1. Wine production (b) 1, 2 and 3

2. Orchard farming (c) 2 and 3 only

3. Cotton cultivation (d) 1 and 2 only

Select the correct answer using the code


5. With reference to anti-conversion laws,
given below.
consider the following statements:
(a) 1 and 3 only
1. Article 25 of the Indian Constitution
(b) 2 and 3 only
restricts conversion of religion.
(c) 1, 2 and 3
2. All states in the country have their anti-
(d) 1 and 2 only
conversion laws in force.
3. Coercive conversion is a penal offence.
3. With reference to dust particles in
Which of the statements given above is/are
atmosphere, consider the following
correct?
statements:
(a) 1 only
1. They are generally concentrated in the
(b) 3 only
lower layers of the atmosphere.
(c) 1 and 2 only
2. They are highly concentrated in
(d) 1, 2 and 3
equatorial and polar regions.
3. They assist in the formation of clouds. 6. When it is 6 PM, Sunday at Greenwich, what
Which of the statements given above are is the time and day at a place 105 degree
correct? East?
(a) 1 and 2 only (a) 1 AM, Monday
(b) 2 and 3 only (b) 11 AM, Sunday
(c) 1 and 3 only (c) 11 AM, Monday
(d) 1, 2 and 3 (d) 1 PM, Sunday
2 www.visionias.in ©Vision IAS

https://www.facebook.com/pdf4exams/
Downloaded From:- https://telegram.me/pdf4exams https://telegram.me/upsc_zone

7. With reference to Tsunami, consider the 11. The four districts of the Union Territory of
following statements: Pondicherry are located in different states.
1. The speed of tsunami wave is more in
Which of the following states is not one of
the shallow water than in the ocean
deep. them?
2. It has long wavelength and short wave- (a) Andhra Pradesh
height in ocean deep than shallow (b) Karnataka
water.
(c) Tamil Nadu
3. It mainly occurs on the eastern coast of
the continents. (d) Kerala
Which of the statements given above are
correct? 12. With respect to the Indian Standard Time
(a) 1 and 2 only
(IST- 82°30’), consider the following
(b) 1 and 3 only
(c) 2 and 3 only statements:
(d) 1, 2 and 3 1. The IST meridian intersects the Tropic
of Cancer in the state of Chhattisgarh.
8. If you travel along the Tropic of Cancer 2. The IST is shared by the neighbouring
(23.5°N) from west to east in India, how
Sri Lanka.
many states would you be crossing?
(a) Five 3. Chaibagaan refers to India's official
(b) Six Daylight Saving Time (DST) used in
(c) Seven north-eastern states.
(d) Eight
Which of the statements given above is/are
correct?
9. The state of Jharkhand does not share
boundary with which of the following states? (a) 1 and 3 only
(a) Uttar Pradesh (b) 1, 2 and 3
(b) Madhya Pradesh
(c) 1 and 2 only
(c) Odisha
(d) Chhattisgarh (d) 2 only

10. Consider the following statements regarding 13. Consider the following statements regarding
mid oceanic ridge: Mohorovicic discontinuity:
1. They occur when convection currents
1. It is the boundary between crust and
rise in the mantle beneath the oceanic
crust mantle
2. They occur where boundary of the 2. Its average depth is same beneath
tectonic plate is divergent oceanic and continental crust.
3. The biggest oceanic ridges are formed
Which of the statements given above is/are
along the subduction zones.
Which of the statements given above is/are correct?
correct? (a) 1 only
(a) 1 only (b) 2 only
(b) 1 and 2 only
(c) Both 1 and 2
(c) 2 and 3 only
(d) 1, 2, and 3 (d) Neither 1 nor 2
3 www.visionias.in ©Vision IAS

https://www.facebook.com/pdf4exams/
Downloaded From:- https://telegram.me/pdf4exams https://telegram.me/upsc_zone

14. The process of ‘lithification’ related with 18. In India, which of the following valley is/are
rocks: rift valley ?
(a) turns sedimentary rocks directly to 1. Son valley
igneous rock 2. Damodar valley
(b) turns raw rock sediment into
3. Mahi valley
consolidated sedimentary rock
Select the correct answer using the code
(c) is responsible for building of block
given below.
mountains
(a) 1 only
(d) turns sedimentary rocks into
metamorphic rocks (b) 1 and 2 only
(c) 2 and 3 only

15. With reference to the geography of India, (d) 1, 2, and 3


Pygmalion Point or Indira point, is the
(a) southernmost point of the country. 19. Consider the following pairs:

T
(b) northernmost point of the country. Islands Location

A
(c) deepest point of the country. 1. Paracel : South China Sea
10 ST
(d) highest point of the country. 2. Hawaii : Atlantic Ocean
3. Diego Garcia : Pacific Ocean
40 TO
16. Which of the following are the
Which of the pairs given above is/are
characteristics of coniferous forests?
correctly matched?
1. The leaves are broad leaved and trees
94 O

(a) 1 and 2 only


have a large spread which provides
55 PH

(b) 1 only
shade during summer months.
2. There is no annual replacement of (c) 2 and 3 only

leaves. (d) 1, 2 and 3


95 .P

3. They can be found in both temperate and


C

tropical regions. 20. Aajeevika Grameen Express Yojana, has


Select the correct answer using the code been recently launched to
given below. (a) provide community monitored rural
(a) 1, 2 and 3 transport services through SHGs to
(b) 2 and 3 only connect remote villages with key
(c) 1 and 2 only
services.
(d) 1 and 3 only
(b) identify poorest of the poor families
within the states for
17. Which one of the following regions of India
providing them foodgrains at a highly
has a combination of montane, moist
subsidized rates.
deciduous and evergreen forests?
(a) North-eastern Hills (c) enable a large number of Indian youth to

(b) Andaman and Nicobar Islands take up industry-relevant skill training.


(c) Eastern Highlands (d) help Panchayat Raj Institutions deliver
(d) Western Himalayas Sustainable Development Goals.
4 www.visionias.in ©Vision IAS

https://www.facebook.com/pdf4exams/
Downloaded From:- https://telegram.me/pdf4exams https://telegram.me/upsc_zone

21. Consider the following statements regarding 24. 'Rakhine state’ of Myanmar borders which
'bays': of the following countries?
1. It is a water body almost surrounded by 1. China
land, with a wide mouth that opens into 2. India
the ocean.
3. Bangladesh
2. It is usually bigger and less enclosed
Select the correct answer using the code
than a gulf.
given below.
Which of the statements given above is/are
(a) 1 and 2 only
correct?
(b) 3 only
(a) 1 only
(b) 2 only (c) 2 and 3 only

(c) Both 1 and 2 (d) 1 only


(d) Neither 1 nor 2
25. Which among the following are evidences of
22. India has recently launched ‘Ocean earth’s sphericity?
Forecasting System’ for which of the 1. Circular Horizon widening with
following countries? increasing altitude
1. Venezuela 2. Shadow cast by earth on moon during
2. Madagascar lunar eclipse
3. Namibia
C.P PHOTOSTAT
Select the correct answer using the code
3. Different timings of sunrise and sunset
at different places
given below. 9555944010
Select the correct answer using the code
(a) 1 and 2 only
given below.
(b) 2 only
(c) 2 and 3 only (a) 1 and 2 only

(d) 1 and 3 only (b) 2 and 3 only


(c) 1 and 3 only
23. Consider the following statements (d) 1, 2 and 3
regarding recently opened Low Enriched
Uranium (LEU) bank: 26. Which among the following are the methods
1. It is owned and controlled by the of soil conservation?
International Atomic Energy Agency. 1. Contour ploughing
2. It would act as a supplier of last resort 2. Terrace farming
for member states in case the supply of
3. Construction of dams
LEU to a nuclear power plant is
4. Strip cropping
disrupted.
Select the correct answer using the code
Which of the statements given above is/are
given below.
correct?
(a) 1 only (a) 1, 2 and 4 only

(b) 2 only (b) 1 and 3 only


(c) Both 1 and 2 (c) 1, 2, 3 and 4
(d) Neither 1 nor 2 (d) 2, 3 and 4 only
5 www.visionias.in ©Vision IAS

https://www.facebook.com/pdf4exams/
Downloaded From:- https://telegram.me/pdf4exams https://telegram.me/upsc_zone

27. Consider the following pairs: 30. Local time of any place is obtained by
Load carried by Material (a) overhead sun at noon
(b) intersection of latitude and longitude
a river
(c) length of degrees of longitude
1. Dissolved load : lime (d) position of pole star
2. Suspended load : pebbles
3. Bedload : silt 31. Which of the following rivers is/are part of
Which of the pairs given above is/are the Arabian Sea drainage?
1. Periyar
correctly matched?
2. Palar
(a) 1 only
3. Penneru
(b) 1 and 3 only Select the correct answer using the code
(c) 2 and 3 only given below.
(d) 2 only (a) 1 only
(b) 1 and 2 only
(c) 2 and 3 only
28. Arrange the following European Rivers in

T
(d) 1, 2 and 3
the direction of West to East:

A
1. Rhine 10 ST 32. Consider the following statements regarding
2. Thames Border Road Organization:
1. It develops and maintain roads in border
3. Danube
40 TO

areas and friendly neighbouring


4. Volga C.P PHOTOSTAT countries.
Select the correct answer using the code 2. It is an organization under Ministry of
9555944010
94 O

given below. Defence.


55 PH

(a) 3-2-1-4 Which of the statements given above is/are


correct?
(b) 2-1-3-4
(a) 1 only
95 .P

(c) 1-2-3-4 (b) 2 only


(d) 2-3-4-1 (c) Both 1 and 2
C

(d) Neither 1 nor 2


29. Which of the following give evidences that
33. Consider the following statements regarding
the Earth has gone through glacial and inter-
epeirogenic forces:
glacial periods?
1. They form continents and oceans.
1. Traces of advances and retreats of 2. Fold formation in the crust is the
glaciers at high latitudes and altitudes predominant feature of epeirogenic
2. Sediment deposits in glacial lakes forces.
3. Under it, rock masses move in vertical
3. Rings in trees
direction to the earth's surface under
Select the correct answer using the code epeirogenic forces.
given below. Which of the statements given above is/are
(a) 2 and 3 only correct?
(a) 1 only
(b) 1 only
(b) 2 only
(c) 1 and 2 only
(c) 1 and 3 only
(d) 1, 2 and 3 (d) None
6 www.visionias.in ©Vision IAS

https://www.facebook.com/pdf4exams/
Downloaded From:- https://telegram.me/pdf4exams https://telegram.me/upsc_zone

34. Which of the following are features of the 37. Consider the following statements:
Cool Temperate Western Margin climate? 1. A strait is a narrow stretch of land
separating two water bodies.
1. Adequate rainfall throughout the year.
2. An isthmus is a narrow passage of water
2. Presence of four distinct seasons.
connecting two water bodies.
3. Mild winters and cool summers. Which of the statements given above is/are
Select the correct answer using the code correct?
given below. (a) 1 only
(a) 1 and 2 only (b) 2 only
(c) Both 1 and 2
(b) 2 and 3 only
(d) Neither 1 nor 2
(c) 1 and 3 only
(d) 1, 2 and 3 38. Which among the following
panels/committee was constituted to give
35. Consider the following statements: recommendation regarding wages paid under
1. There are large changes in the mean MGNREGA?
(a) Kadiyam Srihari panel
temperature over short distances.
(b) Arvind Panagariya committee
2. Precipitation types and its intensity vary
(c) Nagesh Singh panel
spatially. (d) Dinesh Sharma committee
3. There is vertical zonation of climatic
types with elevation. 39. Which of the following is/are responsible for
The above C.P PHOTOSTAT
statements represent the causing floods in India?
1. Cyclone
characteristics9555944010
of which of the following
2. Sediment deposition
types of climates?
3. Deforestation
(a) Cold Snow Forest Climates Select the correct answer using the code
(b) Highland Climates given below.
(c) Dry Climates (a) 1 and 2 only
(d) Polar Climates (b) 1 only
(c) 2 only
(d) 1, 2 and 3
36. With reference to our solar system, consider
the following statements: 40. Consider the following statements about
1. All planets revolve around the sun in limestone and chalk landforms:
circular orbits. 1. These land forms are made up of
2. All the planets have natural satellites. calcium carbonate and magnesium
2. There is an abundance of surface
3. Inner planets have ring systems around
drainage on these landforms
them. 3. Stalactites and stalagmites are
Which of the statements given above is/are underground features of these landforms
correct? Which of the statements given above is/are
(a) 1 and 2 only correct?
(a) 1 only
(b) 3 only
(b) 1 and 2 only
(c) 1 only
(c) 1 and 3 only
(d) None (d) 2 and 3 only
7 www.visionias.in ©Vision IAS

https://www.facebook.com/pdf4exams/
Downloaded From:- https://telegram.me/pdf4exams https://telegram.me/upsc_zone

41. Consider the following statements regarding 44. Which of the following statements regarding
Article 35A of Indian Constitution: temperate and tropical deserts are correct?
1. It was incorporated in the Constitution 1. Cold ocean currents help in the
formation of both temperate and tropical
along with Article 370.
deserts.
2. The Article regulates ownership of
2. The annual range of temperature in
property in the state of Jammu &
temperate desert is higher than tropical
Kashmir.
desert.
Which of the statements given above is/are 3. While the major temperate deserts are
correct? located on the eastern coast of the
(a) 1 only continents, the tropical deserts are
(b) 2 only located on the western coast.
(c) Both 1 and 2 Select the correct answer using the code
given below.
(d) Neither 1 nor 2
(a) 1 and 2 only

T
(b) 2 and 3 only
42. BHARAT 22, recently seen in news is a/an

A
(c) 1, 2 and 3
(a) action plan drafted by Niti Aayog to
10 ST (d) 1 and 3 only
achieve Sustainable development goals
related to health by 2022
40 TO
45. Consider the following statements:
C.P PHOTOSTAT
(b) tourism project for developing thematic 1. Isobars are lines connecting places
circuits throughout the country. having equal temperatures.
9555944010
94 O

(c) emission standards instituted by the 2. The isobars in the northern hemisphere
55 PH

Government of India to regulate the are highly distorted due to large areas
occupied by continents.
output of air pollutants from internal
Which of the statements given above is/are
95 .P

combustion engines.
correct?
(d) exchange traded fund launched to
C

(a) 1 only
facilitate disinvestment by the (b) 2 only
government. (c) Both 1 and 2
(d) Neither 1 nor 2
43. The indigenous NavIC satellite constellation
offers which of the following services? 46. The recently discovered Malpelo tectonic
1. Disaster management plate is located in which of the following
2. Vehicle tracking and fleet management areas?
(a) Off the coast of Ecuador in the eastern
3. Precise timing
Pacific Ocean.
Select the correct answer using the code
(b) Off the south-eastern coast of Africa,
given below.
close to Madagascar.
(a) 2 only (c) Continental location occupying Anatolia
(b) 1, 2 and 3 peninsula.
(c) 1 and 2 only (d) Continental location on Antarctic
(d) None peninsula.
8 www.visionias.in ©Vision IAS

https://www.facebook.com/pdf4exams/
Downloaded From:- https://telegram.me/pdf4exams https://telegram.me/upsc_zone

47. Consider the following pairs: 50. Which of the following countries surround
Layers/spheres Properties Gulf of Thailand?
1. Cambodia
1. Crust : consists of
2. Vietnam
oceanic and 3. Philippines
continental plate 4. Myanmar
2. Lithosphere : consists of crust Select the correct answer using the code
given below.
and upper most
(a) 1, 2, 3 and 4
mantle (b) 1 and 2 only
3. Asthenosphere : a part of lower (c) 1, 3 and 4
(d) 2 and 4 only
mantle

Which of the pairs given above is/are 51. Which of the following cities is/are correctly
correctly matched? matched with their type of climate?
City Climate
(a) 1 only
1. Rome : Meditteranean
(b) 1 and 2 only 2. Sydney : Savannah
(c) 2 and 3 only 3. Kuala Lumpur : Equatorial
(d) 1, 2 and 3 Select the correct answer using the code
given below.
C.P PHOTOSTAT (a) 1 and 3 only
48. Which of the following pairs is/are correctly
matched?
9555944010 (b)
(c)
1only
2and3only
Traditional sports States (d) 1, 2 and 3
1. Archery : Jharkhand
52. Consider the following pairs:
2. Mukna : Manipur
Minor Relief Properties
3. Gatka : Punjab Feature
Select the correct answer using the code 1. Seamount : a mountain with
given below. pointed summits,
rising from the
(a) 2 only
seafloor
(b) 1 and 2 only 2. Submarine : a ring-shaped reef
(c) 1 only canyons
(d) 1, 2 and 3 3. Atoll : deep valleys in
continental shelves
and slopes
49. Doklam plateau is a tri-junction between Which of the pairs given above is/are
(a) China, India and Nepal correctly matched?
(b) China, Nepal and Bhutan (a) 1 only
(b) 1 and 2 only
(c) Bangladesh, India, Bhutan
(c) 2 and 3 only
(d) China, Bhutan, India (d) 1, 2 and 3
9 www.visionias.in ©Vision IAS

https://www.facebook.com/pdf4exams/
Downloaded From:- https://telegram.me/pdf4exams https://telegram.me/upsc_zone

53. Consider the following pairs: 55. With respect to India’s borders with its
Celestial Bodies Description neighbours, consider the following
1. Comet : Rocks orbiting statements:
between Mars 1. India- Myanmar boundary runs along the
watershed between Brahmaputra and
and Jupiter
Irrawady rivers.
2. Asteroid : Rock originating
2. India shares longest international border
in outer solar
with Pakistan.
system
3. India shares boundary with Afghanistan
accompanied by through Wakhan corridor.
a tail Which of the statements given above is/are
3. Meteor : A burning rock correct?
appearing as (a) 1 and 2 only
streak of light in (b) 2 and 3 only
earth’s (c) 1 and 3 only

T
atmosphere (d) 1, 2 and 3

A
Which of the pairs given above is/are
10 ST 56. Consider the following statements:
correctly matched?
1. The midday sun is exactly overhead at
(a) 1, 2 and 3
40 TO

least once a year on all latitudes.


(b) 3 only C.P PHOTOSTAT 2. The angle of sun’s rays goes on
(c) 2 and 3 only
9555944010
94 O

increasing towards the poles.


(d) None of the above Which of the above statements is/are
55 PH

correct?
54. With reference to Intensified Mission (a) 1 only
95 .P

Indradhanush, consider the following (b) 2 only


statements: (c) Both 1 and 2
C

1. It will focus on improving immunization (d) Neither 1 nor 2


coverage through need based
interventions. 57. Consider the following statements regarding
the elements found in earth or earth's interior
2. It will focus on pregnant women who
in terms of weight:
have missed out on routine
1. Silicon content is highest in earth’s crust
immunization.
2. Iron content is highest in the total mass
3. It includes a component of vaccination
of the earth
for children up to 5 years of age. 3. Oxygen content is highest in mantle
Which of the statements given above are Which of the statements given above is/are
correct? correct?
(a) 1 and 2 only (a) 1 only
(b) 2 and 3 only (b) 1 and 2 only
(c) 1 and 3 only (c) 2 and 3 only
(d) 1, 2 and 3 (d) 1, 2 and 3
10 www.visionias.in ©Vision IAS

https://www.facebook.com/pdf4exams/
Downloaded From:- https://telegram.me/pdf4exams https://telegram.me/upsc_zone

58. ICGS Shaurya, recently commissioned in 62. National Pharmaceutical Pricing Authority
Indian Navy, is can cap the prices of which of the following?
(a) a ballistic missile submarine 1. Drugs
(b) a mine counter measures vessel
2. Medical devices
(c) a torpedo launch vehicle
3. Surgical Procedures
(d) an offshore patrol vessel
Select the correct answer using the code

59. Which of the following phenomena may be given below.

associated with temperature inversion? (a) 1 and 2 only


1. Occurrence of dense fogs in winter (b) 2 and 3 only
mornings. (c) 1 and 3 only
2. Accumulation of smoke and dust (d) 1, 2 and 3
particles in lower atmosphere.
Select the correct answer using the code 63. Arrange the following major oceans in order
given below.
of decreasing size:
(a) 1 only
1. Pacific Ocean
(b) 2 only
2. Indian Ocean
(c) Both 1 and 2
(d) Neither 1 nor 2 3. Atlantic Ocean
C.P PHOTOSTAT 4. Arctic Ocean
60. With reference9555944010
to El Nino Modoki, consider Select the correct answer using the code
the following statements: given below.
1. It is characterized by warming in the (a) 1-2-3-4
central tropical Pacific and cooling in
(b) 1-2-4-3
the eastern and western tropical Pacific.
(c) 2-1-3-4
2. It increases the frequency of cyclones
(d) 1-3-2-4
over the Arabian Sea.
Select the correct answer using the code
given below. 64. Which of the following factors influence the

(a) 1 only climate of Europe?


(b) 2 only 1. Westerlies
(c) Both 1 and 2 2. North Atlantic Drift
(d) Neither 1 nor 2 3. Alpine System
Select the correct answer using the code give
61. With respect to the different types of
below.
vegetation, what does Gran Chaco refers to?
(a) 3 only
(a) Warm temperate forests
(b) 2 and 3 only
(b) Tropical grasslands
(c) Mangrove Forests (c) 1, 2 and 3

(d) Equatorial rain-forests (d) 1 and 2 only


11 www.visionias.in ©Vision IAS

https://www.facebook.com/pdf4exams/
Downloaded From:- https://telegram.me/pdf4exams https://telegram.me/upsc_zone

65. Earth Overshoot day, recently seen in news, 68. Which of the following is the correct
is sequence of rivers in decreasing order of
their drainage basin in India?
(a) a movement in which the participants (a) Brahmaputra>Mahanadi>Narmada>Kris
switch off the lights for one hour on a hna
(b) Krishna>Brahmaputra>Mahanadi>Narm
certain day every year.
ada
(b) date when humanity's annual demand on (c) Brahmaputra>Narmada>Mahanadi>Kris
nature exceeds what Earth can hna
(d) Mahanadi>Narmada>Brahmaputra>Kris
regenerate over the entire year.
hna
(c) a day celebrated to raise awareness
regarding depletion of ozone layer. 69. Which of the following climatic regions
(d) none of the above. receives the maximum insolation at the
surface?
(a) Equatorial forests

T
66. Soils of tropical and equatorial climates are (b) Subtropical deserts

A
very low in humus content because of: (c) Sub-polar Ice sheets

1. high precipitation
10 ST (d) Temperate Grasslands

2. low bacterial growth 70. Consider the following statements about


40 TO

C.P PHOTOSTAT
Select the correct answer using the code surface waves:
1. They propogate on the interface of two
given below.
9555944010
94 O

mediums like earth and atmosphere.


(a) 1 only 2. They can travel in solid, liquid and gas
55 PH

(b) 2 only materials.


3. They are last to report on seismograph
(c) Both 1 and 2
and are more destructive.
95 .P

(d) Neither 1 nor 2 Which of the statements given above is/are


C

correct?
67. Which of the following are erosional (a) 1 only
(b) 1 and 2 only
features?
(c) 1 and 3 only
1. Arches (d) 2 and 3 only
2. Moraines
71. Which of the following is/are factors
3. Sand dunes
responsible for condensation?
4. U- shaped valleys 1. presence of hygroscopic nuclei
Select the correct answer using the code 2. low temperature
3. presence of moisture
given below.
Select the correct answer using the code
(a) 1 and 4 only given below.
(b) 2 and 3 only (a) 1 only
(b) 1 and 2 only
(c) 1 and 2 only
(c) 2 and 3 only
(d) 1, 2, 3 and 4 (d) 1, 2 and 3
12 www.visionias.in ©Vision IAS

https://www.facebook.com/pdf4exams/
Downloaded From:- https://telegram.me/pdf4exams https://telegram.me/upsc_zone

72. Consider the following: 75. D.B Shekatkar Panel, recently seen in news,
1. NRIs was constituted for
2. Service voters (a) addressing the issues of corporate
3. Overseas citizen of India governance.
Which of the above is/are eligible to cast (b) disinvestment of shares of PSUs.
their vote through proxy? (c) enhancing the combat capability of army
(a) 1 only and rebalancing defence expenditure.
(b) 1 and 2 only (d) revisiting the Defence Procurement
(c) 2 and 3 only Policy.
(d) 1, 2 and 3

76. Which of the following statements regarding


73. In the geological time scale, arrange the
alluvial soils is/are correct?
given eras from earliest to latest:
1. They cover entire northern plains.
1. Pre Cambrian
2. They are generally rich in potash.
2. Paleozoic
3. They are highly fertile soils due to
3. Mesozoic
richness of humus content.
4. Cenozoic
C.P PHOTOSTAT
Select the correct answer using the code
Select the correct answer using the code
given below.
given below. 9555944010
(a) 1 and 2 only
(a) 4-1-2-3
(b) 2 only
(b) 1-2-3-4
(c) 1 and 3 only
(c) 3-2-4-1
(d) 1, 2 and 3
(d) 1-3-4-2

77. Which among the following programmes


74. With reference to heat budget of the Earth,
have an impact on the Infant Mortality Rate
consider the following statements:
(IMR)?
1. The earth's atmosphere absorbs no heat
from the incoming solar radiation. 1. JananiSurakshaYojana

2. Clouds reflect more incoming radiation 2. Mid Day Meal Scheme

than the earth's cryosphere. 3. RashtriyaBalSwasthyaKaryakram

Which of the statements given above is/are Select the correct answer using the code

correct? given below.

(a) 1 only (a) 1 and 2 only

(b) 2 only (b) 2 and 3 only

(c) Both 1 and 2 (c) 1 and 3 only


(d) Neither 1 nor 2 (d) 1, 2 and 3
13 www.visionias.in ©Vision IAS

https://www.facebook.com/pdf4exams/
Downloaded From:- https://telegram.me/pdf4exams https://telegram.me/upsc_zone

78. With reference to atmospheric carbon 81. With reference to Madhyamik and Uchchtar
dioxide, consider the following statements: Shiksha Kosh (MUSK), consider the
following statements:
1. It is the largest constituent of the Earth's
1. It will be maintained by Ministry of
atmosphere. Human Resource Development.
2. In the atmosphere, it is not found beyond 2. It will form a part of Consolidated Fund
the troposphere. of India.
3. The funds arising from the MUSK
Which of the statements given above is/are
would be utilized for schemes in the
correct? tribal areas only.
(a) 1 only Which of the statements given above is/are
(b) 2 only correct?
(a) 1 only
(c) Both 1 and 2
(b) 1 and 2 only
(d) Neither 1 nor 2 (c) 1 and 3 only

T
(d) 2 and 3 only

A
79. With reference to Isotherms, consider the
following statements:
10 ST 82. Which of the following local and seasonal
winds blow towards the Mediterranean sea
1. During winters in the northern
40 TO
from surrounding areas?
C.P PHOTOSTAT
hemisphere, they deviate towards north 1. Chinook
over the ocean. 2. Mistral
9555944010
94 O

2. In southern hemisphere they almost run 3. Sirocco


55 PH

Select the correct answer using the code


parallel to the latitudes.
given below.
3. They intersect each other at the equator.
(a) 1 and 2 only
95 .P

Which of the statements given above are (b) 2 and 3 only


C

correct? (c) 1 and 3 only


(d) 1, 2 and 3
(a) 1 and 2 only
(b) 2 and 3 only
83. With reference to the tropical cyclone,
(c) 1 and 3 only consider the following statements:
(d) 1, 2 and 3 1. Kerala is less prone to tropical cyclone
as compared to Odisha.
2. Hyderabad is more prone to disaster
80. Ghantasala, a historical place in India,
from tropical cyclone than
recently seen in news, is related to which of
Vishakapatnam.
the following religion? Which of the statements given above is/are
(a) Jainism correct?
(a) 1 only
(b) Buddhism
(b) 2 only
(c) Hinduism
(c) Both 1 and 2
(d) Sikhism (d) Neither 1 nor 2
14 www.visionias.in ©Vision IAS

https://www.facebook.com/pdf4exams/
Downloaded From:- https://telegram.me/pdf4exams https://telegram.me/upsc_zone

84. With respect to Long Term Irrigation Fund, 87. Which of the following are the direct sources
consider the following statements: of information about the interior of Earth?
1. It aims to funding and fast tracking the 1. Ocean Drilling
implementation of incomplete major and 2. Volcanic eruption
medium irrigation projects. 3. Earthquake
2. It has been instituted in NABARD under 4. Magnetic field

Pradhan Mantri Krishi Sinchayee Select the correct answer using the code
given below.
Yojana.
(a) 1, 2, 3 and 4
Which of the statements given above is/are
(b) 1 and 2 only
correct?
(c) 2 and 4 only
(a) 1 only
(d) 3 and 4 only
(b) 2 only
(c) Both 1 and 2
88. Consider the following pairs:
(d) Neither 1 nor 2
River Delta
1. Mississippi : Bird's Foot
85. A large part of the continent is arid and 2. Nile : Arcuate
semi-arid land. It is known for its pastoral 3. Narmada : Lacustrine
industries. It contains the world’s large Which of the pairs given above is/are
artesian system’. correctly matched?
Which of the C.P
following PHOTOSTAT
continents best fits (a) 1 and 2 only
9555944010
the above description? (b) 2 only
(a) South America (c) 2 and 3 only
(b) Australia (d) 3 only
(c) North America
(d) Africa 89. Which among the following processes is
primarily responsible for flooding the
86. While moving in the direction of the East atmosphere with oxygen during the

African Rift Valley from south to north evolution of Earth?


(a) Degassing
direction, which of the following countries
(b) Photosynthesis
will be crossed?
(c) Volcanic eruptions
1. Tanzania
(d) Diastrophism
2. Nigeria
3. Kenya
90. Gully erosion is:
4. Ethiopia
(a) coastal erosion by strong sea waves.
Select the correct answer using the code
(b) soil erosion on flat lands by heavy
given below.
rainfall.
(a) 1, 2, 3 and 4 (c) soil erosion on steep slopes by heavy
(b) 1, 3 and 4 rainfall.
(c) 2 and 3 only (d) soil erosion in arid and semi-arid areas
(d) 1 and 4 only by winds.
15 www.visionias.in ©Vision IAS

https://www.facebook.com/pdf4exams/
Downloaded From:- https://telegram.me/pdf4exams https://telegram.me/upsc_zone

91. Which of the following atmospheric layers 95. Consider the following features of a type of
exhibit increase in temperature with soil:
increasing altitude?
1. It is rich in humus and organic content.
1. Troposphere
2. It is highly acidic.
2. Stratosphere
3. Mesosphere 3. It is widely found in northern part of
4. Thermosphere Bihar and coastal areas of West Bengal.
Select the correct answer using the code Which of the following soils is best
given below.
described by the above features?
(a) 1 and 3 only
(a) Peaty Soil
(b) 2 and 4 only
(c) 1, 2 and 3 only (b) Saline Soil
(d) 2, 3 and 4 only (c) Arid Soil
(d) Yellow Soil
92. Djibouti borders which of the following

T
water bodies?
96. The only active volcano under Indian

A
1. Gulf of Aden
2. Red Sea 10 ST territory was in news recently. Where is that
3. Gulf of Suez volcano situated?
4. Persian Gulf (a) Dhosi Hill
40 TO

Select the correct answer using the code


given below. C.P PHOTOSTAT (b) Narcondam Island
(c) Baratang Island
9555944010
94 O

(a) 1 only
(b) 1 and 2 only (d) Barren Island
55 PH

(c) 2, 3 and 4 only


(d) 1, 2, 3 and 4 97. Ber, neem, babool, date palm are common
95 .P

trees found in which type of vegetation?


93. The terms solution, oxidation, carbonation,
(a) Moist deciduous forests
C

hydration are related to:


(b) Tropical evergreen forests
(a) Physical weathering
(b) Chemical weathering (c) Dry deciduous forests
(c) Biological weathering (d) Tropical desert forests
(d) None of the above
98. Which among the following is responsible
94. The crustal surface is uneven because of:
for variation in length of day and night at
1. variations in geothermal gradients within
different times of the year?
the earth
2. crustal thickness and strength (a) Revolution of the earth and its
Select the correct answer using the code inclination to the plane of the ecliptic
given below. (b) Rotation of the earth on a tilted axis
(a) 1 only
(c) Shape of the earth
(b) 2 only
(d) Presence of more landmass in northern
(c) Both 1 and 2
(d) Neither 1 nor 2 hemisphere
16 www.visionias.in ©Vision IAS

https://www.facebook.com/pdf4exams/
Downloaded From:- https://telegram.me/pdf4exams https://telegram.me/upsc_zone

99. Consider the following statements regarding


Kuroshio Current:
1. It is a cold current in the Western North
Pacific Ocean.
2. It is responsible for sustaining the coral
reefs of Japan.
Which of the statements given above is/are
correct?
(a) 1 only
(b) 2 only
(c) Both 1 and 2
(d) Neither 1 nor 2

100. Consider the following pairs:


Regions Separated by
1. Andaman and : Ten degree
Nicobar Islands channel.
2. Indian mainland : Palk Strait
and Srilanka
3. Laccadive and : Eight degree
Minicoy channel.
Islands

C.P PHOTOSTAT
Which of the pairs given above is/are
correct?
(a) 1, 2 and 39555944010
(b) 1 and 2 only
(c) 1 only
(d) 2 and 3 only

Copyright © by Vision IAS


All rights are reserved. No part of this document may be reproduced, stored in a retrieval system or transmitted
in any form or by any means, electronic, mechanical, photocopying, recording or otherwise, without prior
permission of Vision IAS.

17 www.visionias.in ©Vision IAS

https://www.facebook.com/pdf4exams/
Downloaded From:- https://telegram.me/pdf4exams https://telegram.me/upsc_zone

VISIONIAS
www.visionias.in

ANSWERS & EXPLANATION


GENERAL STUIDES (P) TEST – 2439 (2018)

Q 1.C
Loo is a strong, hot and dry summer afternoon wind from the west which blows over the western Indo-
Gangetic Plain region of North India and Pakistan. It is especially strong in the months of May and
June. In tropical regions particularly in northern India during summer season, local winds called 'loo' are
the outcome of advection process. The transfer of heat through horizontal movement of air is called
advection.

A T
Q 2.D
o
10 ST
Mediterranean climate is ideal for the production of wines, because the best wine is essentially made
from grapes. The long, sunny summer allows the grapes to ripen. Viticulture is by tradition a
Mediterranean occupations and regions bordering the Mediterranean Sea account for three-quarters of the
40 TO
world’s production of wine.
o The Mediterranean lands are known as the world’s orchard lands as climate of the region suitable for
the wide range of citrus fruits. The thick, leathery skin of the citrus fruits prevents excessive transpiration
94 O

and long, sunny summer enables the fruits to be ripened and harvested. Besides this fruit trees have long
roots to draw water from considerable depths during the long summer drought.
55 PH

o Cotton cultivation is chief economic activity of China type climate.

Q 3.C
Atmosphere has a suffient capacity to keep small solid particles, which may originate from different
95 .P

sources and include sea salts, fine soil, smoke-soot, ash, pollen, dust and disintergrated particles of
meteors.
C

Dust particles are generally concentrated in the lower layers of the atmosphere; yet, convectional air
currents may transport them to great heights. The higher concentration of dust particles is found in
subtropical and temperate regions due to dry winds in comparison to equatorial and polar regions. Hence
statement 1 is correct and statement 2 is not correct.
Dust and salt particles act as hygroscopic nuclei around which water vapour condenses to produce
clouds. Hence, statement 3 is correct.

Q 4.B
What is GI tag?
 The GI tag is an indication that is definite to a specific geographical territory. It is used for
agricultural, natural and manufactured goods having special quality and established reputation.
Hence, statement 1 is correct.
 The GI tag is governed by the Geographical Indications of Goods (Registration and Protection
Act), 1999.
 This Act is administered by the Controller General of Patents, Designs and Trade Marks, who is
also the Registrar of Geographical Indications, DIPP, Ministry of Commerce and Industry.
Hence, statement 3 is correct.
 The registration of a GI is valid for 10 years after which it needs to be renewed. Hence,
statement 2 is correct.

1 www.visionias.in ©Vision IAS

https://www.facebook.com/pdf4exams/
Downloaded From:- https://telegram.me/pdf4exams https://telegram.me/upsc_zone

Q 5.B
Art. 25 of the Indian Constitution provides for right to practice, propagate, profess one’s religion. It is
silent on conversion of religion. Hence, statement 1 is not correct.
5 states in the country have anti-conversion laws in force, they are - Odisha, MP, Chhattisgarh, Gujarat,
HP. Hence, statement 2 is not correct.
Forced conversion is a penal offence under Section 295 of the Indian Penal Code. Hence, statement 3 is
correct.
Q 6.A
Time at Greenwich = 6 PM
Destination = 105°E
Conversion of degree to time
1 hour = 15° and 4 minutes = 1°
Hence 105° = 7 hours, that is 1 AM, Monday.
Q 7.A
Earthquakes and volcanic eruptions that cause the sea-floor to move abruptly resulting in sudden
displacement of ocean water in the form of high vertical waves are called tsunamis (harbor waves) or
seismic sea waves. Normally, the seismic waves cause only one instantaneous vertical wave; but, after the
initial disturbance, a series of after waves are created in the water that oscillate between high crest and
low trough in order to restore the water level.
The speed of wave in the ocean depends upon the depth of water. It is more in the shallow water than in
the ocean deep. As a result of this, the impact of tsunami is less over the ocean and more near the coast
where they cause large-scale devastations. Hence, statement 1 is correct.
Therefore, a ship at sea is not much affected by tsunami and it is difficult to detect a tsunami in the deeper
parts of sea. It is so because over deep water the tsunami has very long wave-length and limited wave-
height. Thus, a tsunami wave raises the ship only a meter or two and each rise and fall takes several
minutes. Hence, statement 2 is correct.

Q 8.D C.P PHOTOSTAT


Tsunami is not region or area specific. Hence, statement 3 is not correct.

9555944010
The person would be crossing 8 Indian states, if he travels along the Tropic of Cancer (23.5°N) which are
as follows (from West to East) :
1. Gujarat
2. Rajasthan
3. Madhya Pradesh
4. Chhattisgarh
5. Jharkhand
6. West Bengal
7. Tripura
8. Mizoram

2 www.visionias.in ©Vision IAS

https://www.facebook.com/pdf4exams/
Downloaded From:- https://telegram.me/pdf4exams https://telegram.me/upsc_zone

Q 9.B
Option (b) is the correct answer.
Jharkhand shares border with the following states:
1. West Bengal
2. Bihar
3. Uttar Pradesh
4. Chhattisgarh
5. Odisha
It does not share boundary with Madhya Pradesh.

A T
10 ST
40 TO

C.P PHOTOSTAT
9555944010
94 O
55 PH
95 .P
C

Q 10.B
Uplifting of the ocean floor occurs when convection currents rise in the mantle beneath the oceanic crust
and create magma where two tectonic plates meet at a divergent boundary. Hence, statement 1 and 2 are
correct.
The mid-ocean ridges of the world are connected and form a single global mid-oceanic ridge system that
is part of every ocean, making the mid-oceanic ridge system the longest mountain range in the world, with
a total length of about 60,000 km.
There are two processes, ridge-push and slab-pull, thought to be responsible for the spreading seen at mid-
ocean ridges, and there is some uncertainty as to which is dominant.
Ridge-push occurs when the weight of the ridge pushes the rest of the tectonic plate away from the ridge,
often towards a subduction zone. At the subduction zone, 'slab-pull' comes into effect. This is simply the
weight of the tectonic plate being subducted (pulled) below the overlying plate dragging the rest of the
plate along behind it. Hence, statement 3 is not correct, as subduction zone is destroying the plate not
creating it.
The other process proposed to contribute to the formation of new oceanic crust at mid-ocean ridges is the
'mantle conveyor'.
However, there have been some studies which have shown that the upper mantle (asthenosphere) is too
plastic (flexible) to generate enough friction to pull the tectonic plate along.

3 www.visionias.in ©Vision IAS

https://www.facebook.com/pdf4exams/
Downloaded From:- https://telegram.me/pdf4exams https://telegram.me/upsc_zone

Q 11.B
Option (b) is the correct answer.
Puducherry, original name Putucceri, formerly (until 2006) Pondicherry, also spelled Pondichérry, union
territory of India. It was formed in 1962 out of the four former colonies of French India: Pondicherry
(now Puducherry) and Karaikal along India’s southeastern Coromandel Coast, surrounded by Tamil Nadu
state; Yanam, farther north along the eastern coast in the delta region of the Godavari River, surrounded
by Andhra Pradesh state; and Mahe, lying on the western Malabar Coast, surrounded by Kerala state. The
territory’s capital is the city of Puducherry in the Puducherry sector, just north of Cuddalore.
• The Karaikal sector, south of the Puducherry sector, is in the fertile KaveriRiverdelta, in one of the most
important rice-producing areas of India. The exceptional fertility of the region is to some extent reflected
in the unusually high density of its rural population. The town is on the Mayavaram-Peralam route, a
branchline of the southern railway.
• The Mahe sector consists of two parts: the quaint picturesque town of Mahe, with its buildings situated
on the left bank of the Mahe River close to its mouth; and the isolated tract known as Naluthrara, on the
right bank, comprising the four villages of Chambara, Chalakara, Palour, and Pandaquel. Rice is the chief
crop grown in the sector.
• Yanam is a small town on the bank of a branch of the Godavari River, about 400 miles (650 km) north
of the city of Chennai (Madras), near Kakinada.
The major languages spoken in the areas are Tamil, Malayalam, and Telugu. Tamil is predominant in the
southern settlements of Puducherry and Karaikal; Malayalam is predominant in Mahe; and Telugu is
spoken mainly in Yanam. Other significant languages in the territory include Urdu, French, Kannada,
Hindi, Gujarati, English, and Marathi.

C.P PHOTOSTAT
9555944010

Q 12.D
 Statement 1 is not correct: Established on September 1, 1947, IST corresponds to the time schedule
along the 82.5°E longitude near Mirzapur in Uttar Pradesh. States located to the west of this longitude
have more daylight hours compared to those in the east. Indian Standard Time is five and a half hours
ahead of UTC / GMT.
 Statement 2 is correct: The Sri Lanka Time reverted on 15 April 2006 to match Indian Standard Time
calculated from the Allahabad Observatory in India 82.5 ° longitude East of Greenwich, the reference
point for GMT.
 Statement 3 is not correct. India has no official Daylight Saving Time (DST) although the country spans
more than 2000 Km East to West. Some of the North eastern states have a different 'Tea Garden Time' to
save Daylight. Chaibagaan or bagaan time was set one hour ahead of IST for tea estates, collieries and oil
industry of Assam. Most tea estates still follow this time, so does the 112-year-old Digboi Refinery in
eastern Assam.
4 www.visionias.in ©Vision IAS

https://www.facebook.com/pdf4exams/
Downloaded From:- https://telegram.me/pdf4exams https://telegram.me/upsc_zone

Q 13.A
o Statement 1 is correct: The Mohorovicic Discontinuity, or 'Moho,' is the boundary between the crust and
the mantle. In geology the word 'discontinuity' is used for a surface at which seismic waves change
velocity. One of these surfaces exists at an average depth of 8 kilometers beneath the ocean basin and at
an average depth of about 32 kilometers beneath the continents. At this discontinuity, seismic waves
accelerate. This surface is known as the MohorovicicDiscontinuity or often simply referred to as the
'Moho.'

A T
o Statement 2 is not correct: This seismic discontinuity is now know as the Moho (much easier
than 'Mohorovicic seismic discontinuity') It is the boundary between the felsic/mafic crust with seismic
10 ST
velocity around 6 km/sec and the denser ultramafic mantle with seismic velocity around 8 km/sec. The
depth to the Moho beneath the continents averages around 35 km but ranges from around 20 km to 70 km.
The Moho beneath the oceans is usually about 7 km below the seafloor (i.e., ocean crust is about 7 km
40 TO

thick).
C.P PHOTOSTAT
Q 14.B
9555944010
94 O

 All sedimentary rocks are lithified into some collective mass. Lithification is any process that turns raw
55 PH

rock sediment into consolidated sedimentary rock. The process of lithification usually produces
identifiable layering in these type of rocks. Lithification can occur by way of:
o Drying and compaction.
o Oxidation of iron and aluminum.
95 .P

o Precipitation of calcium and silica.


 Lithification is complex process whereby freshly deposited loose grains of sediment are converted
C

into rock. Lithification may occur at the time a sediment is deposited or later. Cementation is one of the
main processes involved, particularly for sandstones and conglomerates. In addition, reactions take place
within a sediment between various minerals and between minerals and the fluids trapped in the pores;
these reactions, collectively termed authigenesis, may form new minerals or add to others already present
in the sediment.
 The difference between lithification and metamorphism is that lithification is (geology) the compaction
and cementation of sediment into rock while metamorphismis (geology) the process by which rocks are
changed into other forms by the application of heat and/or pressure.

Q 15.A
Option (a) is the correct answer.
 The southernmost point of the country is the Pygmalion Point or Indira Point and is located at 6° 45′
N latitude.
 It is a village in the Nicobar district of Andaman and Nicobar Islands, India.
 The Indira Point has a light house which establishes the true sense of the term point for the
destination. It is one of the most important points for trade ships, naval ships and even local fishing
ships that are guided by it day in and out.
 It is largely covered by rainforests comprising of a variety of wildlife.

5 www.visionias.in ©Vision IAS

https://www.facebook.com/pdf4exams/
Downloaded From:- https://telegram.me/pdf4exams https://telegram.me/upsc_zone

Q 16.B

C.P PHOTOSTAT
Coniferous trees consist of the four major species- Pine, Fir, Spruce and Larch.
Coniferous forests have the following characteristics:
The trees are conical in shape with sloping branches to adapt to the sub-arctic climate.
9555944010
The leaves are small, thick, leathery and needle-shaped to prevent excessive transpiration.
Hence, statement 1 is not correct.
Almost all conifers are evergreen. The conifer has a two-year fructification cycle. There is no annual
replacement of leaves as in deciduous trees. The same leaf remains on the tree for as long as five
years. Hence, statement 2 is correct.
Coniferous forests are found in other climatic regions where altitude reduces the temperature. The
conifers are dominant trees in mountainous districts of both temperate and tropical countries. Hence,
statement 3 is correct.

Q 17.A
o Hilly regions of north-eastern states have combination of montane forests, moist deciduous forests
and evergreen forests.
o Montane forests are found between altitude of 1000-2000m in higher hilly ranges of northeastern India, as
evergreen broad leaf trees in which oak and chestnut are predominant.
o Moist deciduous forests are more pronounced in the regions which record rainfall between 10-200 cm.
These forests are found in the northeastern states along the foothills of Himalayas as well as eastern
Himalayas, eastern slopes of the Western Ghats and Orissa.
o Tropical evergreen forests are found in the western slope of the Western Ghats, hills of the northeastern
region and Andaman and Nicobar Islands. They are found in warm and humid areas with an annual
precipitation of over 200 cm and mean annual temperature above 22 degrees.
o Hence, option (a) is correct.

Q 18.D
o A rift valley is a lowland region that forms where Earth’s tectonic plates move apart, or rift. Rift valleys
are found both on land and at the bottom of the ocean, where they are created by the process of seafloor
spreading. Rift valleys differ from river valleys and glacial valleys in that they are created by tectonic
activity and not the process of erosion.
6 www.visionias.in ©Vision IAS

https://www.facebook.com/pdf4exams/
Downloaded From:- https://telegram.me/pdf4exams https://telegram.me/upsc_zone

o In India, Son valley, Damodar valley, Tapi valley, Narmada valley are example of rift valley. Hence
option (d) is correct.
o The Son valley is geologically almost a continuation of that of the Narmada River to the southwest. It is
largely forested and sparsely populated. The valley is bordered by the Kaimur Range to the north and
the Chota Nagpur plateau to the south. The river’s flow is seasonal, and the Son is unimportant for
navigation. Dams have been constructed on some of its tributaries.
o Damodar River, river in northeastern India, rising with its many tributaries, notably the Bokaro and
Konar, in the Chota Nagpur plateau of south-central Bihar state. It follows a generally eastward course for
368 miles (592 km) through West Bengal to join the Hugli (Hooghly) River southwest
of Kolkata (Calcutta).
o The Damodar valley along the Bihar–West Bengal border includes India’s most important coal- and mica-
mining fields and has long been an area of active industrial development. The major coalfields (Jharia,
Raniganj, and Giridih) are mostly open-pit and are easily mined.

Q 19.B
Paracel Islands are located in the South China Sea and are among the disputed islands there. Hence, pair
1 is correctly matched.
Hawaii Islands are in the North Pacific Ocean.
Diego Garcia Islands are in the Indian Ocean. Hence, pairs 2 and 3 are not correctly matched.

T
Q 20.A

A
The Government of India has decided to launch a new sub-scheme named ―AajeevikaGrameen Express
Yojana (AGEY)‖ as part of the DeendayalAntyodayaYojana – National Rural Livelihoods Mission
10 ST
(DAY-NRLM). The Self Help Groups under DAY-NRLM will operate road transport service in backward
areas. This will help to provide safe, affordable and community monitored rural transport services to
40 TO
connect remote villages with key services and amenities (such as access to markets, education and health)
C.P PHOTOSTAT
for the overall economic development of backward rural areas. This will also provide an additional avenue
of livelihood for SHGs.

9555944010
94 O

Q 21.A
55 PH

Bay :
o A bay is a small body of water that is set off from a larger body of water generally where the land
curves inward. Hence, statement 1 is correct.
o In simple words, bay is a water body surrounded on three sides by land with the fourth side (mouth) wide
95 .P

open towards oceans. (In Gulfs, the mouth is narrow).


o A bay is usually smaller and less enclosed than a gulf. Hence, statement 2 is not correct.
C

o Example: The Bay of Pigs (Cuba), Hudson Bay (Canada), Bay of Bengal etc.

Gulf : A gulf is a large body of water, sometimes with a narrow mouth, that is almost completely
surrounded by land.

7 www.visionias.in ©Vision IAS

https://www.facebook.com/pdf4exams/
Downloaded From:- https://telegram.me/pdf4exams https://telegram.me/upsc_zone

Q 22.B
The Indian National Centre for Ocean Information Services (INCOIS) of the Ministry of Earth Sciences
here inaugurated the Ocean Forecasting System for Comoros, Madagascar, and Mozambique at the
third Ministerial Meeting of Regional Integrated Multi-Hazard Early Warning System for Asia and Africa
(RIMES), held at Port Moresby, Papua New Guinea.
C.P PHOTOSTAT
The ocean forecast and early warming information on high wave, currents, winds, tides, sub-surface ocean
conditions cater to users like fishermen, coastal population, tourism sector, coastal defence officials,
9555944010
marine police, port authorities, research institutions and offshore industries of these countries.
These ocean services are aimed towards safety at the sea. The system would offer oil spill advisory
services, high wave alerts, port warnings, forecast along the ship routes in addition to tsunami and storm
surge warnings and help in search and rescue operations.

Q 23.C
IAEA is the world's central intergovernmental forum for scientific and technical co-operation in the
nuclear field. International Atomic Energy Agency (IAEA) has opened a uranium bank for Low Enriched
Uranium (LEU) in Oskemen city of Kazakhstan.
Owned and controlled by the IAEA, the LEU Bank will host a reserve of LEU, and act as a supplier
of last resort for Member States in case the supply of LEU to a nuclear power plant is disrupted due
to exceptional circumstances and the Member State is unable to secure LEU from the commercial
market or by any other means.
The IAEA must specify that the LEU can only be used to make fuel for a power reactor. It cannot be used
for the manufacture of any nuclear weapon or nuclear explosive device, or for research and development
of any nuclear weapon or nuclear explosive device, or in such a way as to further any military purpose.
The LEU cannot be enriched further, reprocessed, retransferred, or re-exported unless specifically agreed
to by the IAEA. All relevant IAEA safeguards, safety standards and physical protection measures must
apply to the LEU at all times. Hence, statements 1 and 2 are correct.

Q 24.B
o Rakhine state is a state in Myanmar (Burma). Situated on the western coast, it is bordered by Chin State to
the north, Magway Region, Bago Region and Ayeyarwady Region to the east, the Bay of Bengal to the
west, and the Chittagong Division of Bangladesh to the northwest.
o The Rohingya - a stateless mostly Muslim minority group - have faced years of persecution in Myanmar.
Deep-seated tensions between them and the majority Buddhist population in Rakhine have led to deadly
communal violence in the past.

8 www.visionias.in ©Vision IAS

https://www.facebook.com/pdf4exams/
Downloaded From:- https://telegram.me/pdf4exams https://telegram.me/upsc_zone

o A fresh outbreak of violence in Myanmar's Rakhine state has caused hundreds of thousands of Rohingya
civilians to flee to Bangladesh.

Q 25.D
One of the oldest proofs of the Earth's shape can be seen from the ground and occurs during every lunar
eclipse. The geometry of a lunar eclipse has been known since ancient Greece. When a full Moon occurs

T
in the plane of Earth's orbit, the Moon slowly moves through Earth's shadow. Every time that shadow is

A
seen, its edge is round. Once again, the only solid that always projects a round shadow is a sphere. Hence,
statement 2 is correct. 10 ST
The distant horizon viewed from the deck of a ship at sea is always circular in shape. The circular horizon
widens with increasing altitude and could only be seen on a spherical body. Hence, statement 1 is
correct.
40 TO

The sun rises and sets at different times at different places. If the earth was like a table top, each and every
C.P PHOTOSTAT
place would have sunrise and sunset at the same time, but since our earth is spherical and rotates west to
east, the place located in east receives sunrise and sunset earlier than the places in the west. Hence,
9555944010
94 O

statement 3 is correct.
55 PH

Q 26.C
Soil conservation is a methodology to maintain soil fertility, prevent soil erosion and exhaustion and
improve the degraded condition of the soil. There are the different methods of soil conservation such as
95 .P

1. Terrace farming: On hilly slopes terrace act as buds prevent the soil from being washed away by
running water.
C

2. Contour ploughing: Ploughing along contours of the slopes prevent soils being washed away by
rainwater or by surface runoff.
3. Plugging gullies: Gullies made in soil due water erosion should be plugged with deposition of silt
during heavy rains.
4. Construction of dams: Rivers cause soil erosion. Dams are built in upper course of rivers to control
the erosion of soil.
Other methods of soil conservation are shelter belts, strip cropping, planting trees. Hence option (c) is
correct.

Q 27.A
The amount of loose weathered and eroded rock material carried by a river is called its load. The amount
of load which a river can carry depends on the volume of water and steepness of the slope over which it
flows. Some materials are carried by a river in dissolved form e.g., salt and lime; clay and silt are
carried as suspended load; coarse and heavy materials like pebbles roll and are dragged along the
river bed as bedload.

Q 28.B
The correct sequence
1. Thames
2. Rhine
9 www.visionias.in ©Vision IAS

https://www.facebook.com/pdf4exams/
Downloaded From:- https://telegram.me/pdf4exams https://telegram.me/upsc_zone

3. Danube
4. Volga

C.P PHOTOSTAT
9555944010
Q 29.D
The planet earth has witnessed many variations in climate since the beginning. Geological records show
alteration of glacial and inter-glacial periods.
The geomorphological features, especially in high altitudes and high latitudes, exhibit traces of advances
and retreats of glaciers. Hence, statement 1 is correct.
The sediment deposits in glacial lakes also reveal the occurrence of warm and cold periods. Hence,
statement 2 is correct.
The rings in the trees provide clues about wet and dry periods. Hence, statement 3 is correct.
All these evidences indicate that change in climate is a natural and continuous process.

Q 30.A
Local time of any place is obtained by overhead sun at noon. Every place at a different longitude will
have its own local time. For example, if Mumbai is 73 degree east, and Delhi 77 degree east, it must be 16
minutes past noon at New Delhi, at the same time.

Q 31.A
Periyar River, river in southern Kerala state, southwestern India.Periyar is also the name given to a lake in
the river’s course. The river, 140 miles (225 km) long, rises in the Western Ghats range near the border
with Tamil Nadu state and flows north a short distance to Periyar Lake. The river continues northwest,
descending out of the mountains and onto the coastal plain, and turns west before emptying into the
Arabian sea about 15 miles (24 km) north of Kochi.
Palar River, river in southern India. It rises near the Ponnaiyar River, southwest of Chintamani, in
Karnataka state, and flows 183 miles (295 km) southeastward through Tamil Nadu state to the Bay of
Bengal, south of Chennai (Madras). Its major tributaries are the Ponnai and Cheyyar rivers. Recently, the

10 www.visionias.in ©Vision IAS

https://www.facebook.com/pdf4exams/
Downloaded From:- https://telegram.me/pdf4exams https://telegram.me/upsc_zone

river was also in news, as after a span of nearly 20 years, river Palar has come alive with copious water
flowing from the point where it enters Tamil Nadu at Pullur in Vellore district.
The Penneru rises in an upland region on the Deccan plateau, 7 miles (11 km) west-southwest of
ChikBallapur in southeastern Karnataka. It flows north into Andhra Pradesh state and turns east and then
southeast. After passage through a gap in the Eastern Ghats range, it again bends east toward the
Coromandel Coast, emptying into the Bay of Bengal near Nellore. The river is seasonal, becoming a
torrent after the rains and a thin stream during dry periods.

Q 32.C
o BRO was formed in 1960 with the objective of developing means of communication to the remote areas
of north and north-east. It develops and maintains road networks in India's border areas as well as in
friendly neighboring countries also such as Afghanistan, Bhutan, Myanmar, and Sri Lanka.
o Role of the BRO
In Peace : develop & maintain the Operational Road Infrastructure of General Staff in the Border
Areas; Contribute to the Socio-Economic Development of the Border States.
In War : to develop & maintain roads to keep Line of Control through in Original Sectors and Re-
Deployed Sectors; to execute additional Tasks as laid down by the Govt Contributing to the War Effort.
o Projects taken up by the BRO includes developing roads, bridges, and airfields in hostile environments.
o In a bid to boost border connectivity, the Border Roads Organisation has been entirely brought under
the Ministry of Defence in 2015. Earlier it received funds from the Ministry of Road Transport and

T
Highways.

A
o Recently, Ministry of Defence has decided to delegate administrative and financial powers to the Border
Roads Organisation (BRO) right up to the level of Chief Engineer and Task Force Commander.
10 ST
Q 33.C
40 TO
o Epeirogenic forces are the part of Diastrophic (large scale) forces along with orogenic forces. These forces
C.P PHOTOSTAT
works vertically and horizontally to create various reliefs on the surface of the earth. The difference
between these forces are as follows-

9555944010
1. Orogeny is a mountain building process while epeirogeny is a continent building process. Hence,
94 O

statement 1 is correct.
55 PH

2. In orogeny, crust is severely deformed into folds and it is predominant feature, while in epeirogeny
there may be simple deformation . Hence, statement 2 is not correct.
3. In orogeny, the rock masses move in tangential direction to the earth's surface while in epeirogeny
the rock masses move in radial or vertical direction to the earth's surface. Hence, statement 3 is
95 .P

correct.
4. In orogeny elevated mountains are formed which increases earth’s albedo whereas in epeirogeny sea
C

level rise and fall as new plate materials modify the shape of ocean basins.

Q 34.D
All the options are correct.
The cool temperate western margins are under the permanent influence of the Westerliesall round the
year. They are also regions of much cyclonic activity, typical of Britain and are thus said to experience the
British type of climate.
British Type Climate
 Moderately warm summers and fairly mild winters.
 Rainfall occurs throughout the year with winter maxima.
Temperature
 The mean annual temperatures are usually between 5° C and 15° C.
 Winters are abnormally mild. This is because of the warming effect brought by warm North Atlantic
Drift.
 Sometimes, unusual cold spells are caused by the invasion of cold polar continental air (Polar Vortex)
from the interiors.
Precipitation
 The British type of climate has adequate rainfall throughout the year with a tendency towards a slight
winter maximum (due to frontal cyclones).
 Western margins have the heaviest rainfall due to westerlies.
11 www.visionias.in ©Vision IAS

https://www.facebook.com/pdf4exams/
Downloaded From:- https://telegram.me/pdf4exams https://telegram.me/upsc_zone

 Relief can make great differences in the annual amount. This is particularly significant in New
Zealand where the western margins are subjected to heavy orographic rainfall whereas the eastern
Canterbury plains receive comparatively less rainfall due to rain-shadow effect.
The seasons
 As in other temperate regions there are four distinct seasons.
 Winter is the season of cloudy skies, foggy and misty mornings, and many rainy days from the
passing depressions.
 Spring is the driest and the most refreshing season when people emerge from the depressing winter to
see everything becoming green again.
 This is followed by the long, sunny summer.
 Next is the autumn with the roar of gusty winds; and the cycle repeats itself.
 This type of climate with its four distinct seasons is something that is conspicuously absent in the
tropics. [Rainforest- Only Rainy season, Tropical Monsoon- Summer, Winter and Rainy, Tropical
Savanna- Summer (rains) and Winter.

Q 35.B
Highland climates are governed by topography. In high mountains, large changes in mean temperature
occur over short distances. Precipitation types and intensity also vary spatially across high lands. There is
vertical zonation or layering of climatic types with elevation in the mountain environment.
Hence, (b) is the correct option.

Q 36.D
o All planets revolve around the sun in a fixed path called orbit. Planets revolve round the sun in an
elliptical orbit and not in a circular orbit. Hence, statement 1 is not correct.
o All the planets except Mercury and Venus have natural satellites called moon. Hence, statement 2 is not
correct.
o C.P PHOTOSTAT
The outer planets or the Jovian planets are of gaseous origin and have ring system around them. Hence,
statement 3 is not correct.

Q 37.D
9555944010
Strait : A strait is a narrow waterway that connects two large water bodies. Hence, statement 1 is not
correct.

12 www.visionias.in ©Vision IAS

https://www.facebook.com/pdf4exams/
Downloaded From:- https://telegram.me/pdf4exams https://telegram.me/upsc_zone

An isthmus is a narrow strip of land that connects two larger landmasses and separates two bodies of
water. Hence, statement 2 is not correct.
e.g. Isthmus of Panama lying between the Caribbean Sea and the Pacific Ocean, linking North and
South America

A T
10 ST
40 TO

Q 38.C
C.P PHOTOSTAT
Nagesh Singh panel gave recommendations regarding wages paid under MGNREGA. It recommended:
o There is no need to maintain parity in minimum wages paid by various states under MGNREGA.
9555944010
94 O

o Divergence in pay: Disparity is found between the minimum wages and NREGA wages due to
Minimum Wages which are fixed by the states which arbitrarily increased it without following any
55 PH

scientific principles.
o Panel recommended to switch over to CPI-R from CPI-AL for wage calculation.
95 .P

Q 39.D
The causes of flood in India are as follows:
C

(i) Heavy rainfall: Heavy rain in the catchment area of a river causes water to over flow its banks, which
results in the flooding of nearby areas.
(ii) Sediment deposition: River beds become shallow due to sedimentation. The water carrying capacity
of such river is reduced. As a result the heavy rain waters over flows the river banks.
(iii) Deforestation: Vegetation hampers the flow of water and forces it to percolate in the ground. As a
result of deforestation, the land becomes obstruction free and water flows with greater speed into the
rivers and causes flood.
(iv) Cyclone: Cyclone generated sea waves of abnormal height spreads the water in the adjoining coastal
areas. In October 1994 Orissa cyclone generated severe floods and caused unprecedented loss of life and
property.
(v) Interference in drainage system: Drainage congestion caused by badly planned construction of
bridges, roads, railway tracks, canals etc. hampers the flow of water and the result is flood.
(vi) Change in the course of the river: Meanders and change in the course of the river cause floods.
(vii)Tsunami: Large coastal areas are flooded by rising sea water, when a tsunami strikes the coast.
Hence, all options are correct.

Q 40.C
o In pure state, limestone is made up of calcite or calcium carbonate, but where magnesium is also present it
is termed as dolomite. Hence, statement 1 is correct.

13 www.visionias.in ©Vision IAS

https://www.facebook.com/pdf4exams/
Downloaded From:- https://telegram.me/pdf4exams https://telegram.me/upsc_zone

o Karst topography dominated by sinkholes or dolines usually has several distinct surface features. These
sinkholes or shallow basins may fill with water forming lakes or ponds. Springs are locations where
ground water emerges at the surface of the earth. Disappearing streams are streams which terminate
abruptly by flowing or seeping into the ground. Disappearing streams are evidence of disrupted surface
drainage and thus indicate the presence of an underground drainage system. Hence, statement 2 is not
correct.
o Away from their entrances, caves usually provide a relatively constant temperature and humidity over a
long period of time. Thus, caves provide an ideal environment for chemical deposition of minerals. As
water laden with dissolved carbonate seeps into the air-filled cave passage, it may lose excess carbon
dioxide to the cave atmosphere, or the water itself may evaporate, causing the dripwater to precipitate
secondary carbonate or other minerals from solution, creating cave formations or speleothems including
cone-shaped stalactites, stalagmites, flowstone or rimstone, or other interesting shapes. Caves in karst
areas often have stalactites (icicle-like masses of chemical limestone) that hang from cave ceilings and
stout stalagmites protruding from the cave floor. Stalactites and stalagmites can be a few inches to several
feet long. Hence, statement 3 is correct.

Q 41.B
Article 35A has become the focal point of debate over the special status of Jammu and Kashmir. While
Article 370 remains unchallenged, a Supreme Court bench is examining the constitutional validity of
Article 35A.
Article 35A of the Indian Constitution was incorporated in 1954 through a Presidential Order issued under
A. 370 (1)(d). Hence, statement 1 is incorrect.
The Article empowers the J&K legislature to define state’s permanent residents and special rights and
privileges. It protects certain provisions of the J&K Constitution which denies property rights to native
women who marry from outside the State. Hence, statement 2 is correct.
Q 42.D
Government of India is set to launch its second ETF (Exchange Traded Fund) Bharat 22 which will
C.P PHOTOSTAT
comprise 22 stocks including those of central public sector enterprises (CPSEs), public sector banks and
GOI’s holdings under the Specified Undertaking of Unit Trust of India (SUUTI).
9555944010
The new ETF will help government sell equity stakes in state-run firms and also help in achieving its
objective to raise Rs. 72,500 crore through disinvestment in the current financial year.
Q 43.B
All the options given are correct.
IRNSS is an independent regional navigation satellite system being developed by India. It is designed to
provide accurate position information service to users in India as well as the region extending up to 1500
km from its boundary, which is its primary service area. An Extended Service Area lies between primary
service area and area enclosed by the rectangle from Latitude 30 degSouth to 50 deg North, Longitude 30
deg East to 130 deg East.
IRNSS will provide two types of services, namely, Standard Positioning Service (SPS) which is provided
to all the users and Restricted Service (RS), which is an encrypted service provided only to the authorised
users. The IRNSS System is expected to provide a position accuracy of better than 20 m in the primary
service area.
Some applications of IRNSS are:
 Terrestrial, Aerial and Marine Navigation
 Disaster Management
 Vehicle tracking and fleet management
 Integration with mobile phones
 Precise Timing
 Mapping and Geodetic data capture
 Terrestrial navigation aid for hikers and travellers
 Visual and voice navigation for drivers
Q 44.A
o Statement 1 is correct: Temperate deserts are in the interior of large land mass on the leeward side of
mountains and rainfall bearing winds cannot reach there, this contributes to the formation of these deserts.
Apart from this, cold ocean currents also helps in the formation of temperate desert for instance, cold
Falkland current off the Atlantic coast of South America gives aridity to the Patagonian Desert. On the
14 www.visionias.in ©Vision IAS

https://www.facebook.com/pdf4exams/
Downloaded From:- https://telegram.me/pdf4exams https://telegram.me/upsc_zone

other hand, tropical deserts have less than 25cm rainfall. Since these deserts are located on western
margins of the continents, trade winds that blow in the region shed their moisture in the eastern margins of
continents. They become dry by the time these winds reach western side. Cold ocean currents also help
in formation of tropical deserts as they do not cause rain, mist and fog and lower the temperature.
o Statement 2 is correct: Temperate deserts differ from those at lower latitude in the sense that they have
far greater annual temperature range and much lower winter temperature.
o Statement 3 is not correct: Majority of mid-latitude deserts (Temperate deserts) are found on plateaux
and are at considerable distance from the sea. On the other hand, major hot deserts of the world are
located on the western coasts of continents between latitudes 15 and 30 degree North and South
respectively.

Q 45.B
Horizontal distribution of pressure is studied by drawing isobars at constant levels.
Isobars are lines connecting places having equal pressure. Hence, statement 1 is not correct.
The high pressure of the high latitudes in the northern hemisphere are a little complicated by the presence
of much land, whereas those of the southern hemisphere are straight due to presence of large water
masses. Hence, statement 2 is correct.

Q 46.A

T
Option (a) is the correct answer.
The Malpelo Plate, named after an island and an underwater ridge it contains, is the 57th plate to be

A
discovered and the first in nearly a decade. The plate is discovered off the coast of Ecuador in the eastern
10 ST
Pacific Ocean.
Significance
 The plate movements may explain a mysterious series of very deep, large earthquakes known as the
40 TO
Vityaz earthquakes, which originated in the mantle between Fiji and Australia.
C.P PHOTOSTAT
 With the Malpelo accounted for, the new circuit still doesn't close to zero and the shrinking Pacific
Plate isn't enough to account for the difference either hence laying the ground for probable discovery
9555944010
94 O

of Plate number 58.


Q 47.B
55 PH

o Crust is the outermost solid part of the earth. It is brittle in nature. The thickness of the crust varies
under the oceanic and continental areas. Oceanic crust is thinner as compared to the continental crust.
The mean thickness of oceanic crust is 5 km whereas that of the continental is around 30 km. The
continental crust is thicker in the areas of major mountain systems. It is as much as 70 km thick in the
95 .P

Himalayan region. It is made up of heavier rocks having density of 3 g/cm3 . This type of rock found in
the oceanic crust is basalt. The mean density of material in oceanic crust is 2.7 g/cm3. Hence, pair 1 is
C

correctly matched.
o The crust and the uppermost part of the mantle are called lithosphere. Its thickness ranges from 10-200
km. The lower mantle extends beyond the asthenosphere. It is in solid state. Hence, pair 2 is correctly
matched.
o The portion of the interior beyond the crust is called the mantle. The mantle extends from Moho’s
discontinuity to a depth of 2,900 km. The upper portion of the mantle is called asthenosphere. The
word astheno means weak. It is considered to be extending upto 400 km. It is the main source of magma
that finds its way to the surface during volcanic eruptions. It has a density higher than the crust’s (3.4
g/cm3 ).
The asthenosphere boundary is primarily defined by temperature, and it starts when the crust heats up to
1300° C. At this temperature the crust begins to melt and move more as a liquid. The asthenosphere is
important in plate tectonics, as convection currents slowly move the tectonic plates that lie above. Hence,
pair 3 is not correctly matched.
Q 48.D
In order to promote and develop indigenous games disciplines across the country, Sports Authority of
India (SAI) is promoting Indigenous Games and Martial Arts (IGMA) in 9 disciplines under its National
Sports Talent Contest (NSTC) scheme where talented children are selected in the age group of 8-14 years
in 10 SAI adopted Centers. Government is promoting it through an exclusive component called
―Promotion of Rural, Indigenous and Tribal Games‖. SAI has adopted the following indigenous games
disciplines for their promotion:
15 www.visionias.in ©Vision IAS

https://www.facebook.com/pdf4exams/
Downloaded From:- https://telegram.me/pdf4exams https://telegram.me/upsc_zone

 Kalaripayatu – It is a martial art form which originated in Kerala, originally from northern and central
parts of Kerala and southern Tamil Nadu.
 Silambam – It is a weapon based martial art practiced in Tamil Nadu. Bamboo staff is used as
weapon.
 Archery - It is a sport from Jharkhand in which a bow and arrow are used. Traditionally Archery
was practiced for hunting and recreational purposed.
 Kabaddi – This is a team sport in which two teams compete to remain outnumbered till the end. This
game is widely played in Telangana, Andhara Pradesh and Maharashtra.
 Malkhamb – This traditional sport is a combination of acrobatics and aerial yoga. It is performed on a
wooden pole and the player demonstrates wrestling grip throughout the performance.
 Mukna – It is a form of folk wrestling from Manipur.
 Thangta – It is a martial art form from Manipur.
 Khomlainai – It is a martial art performed by the Bodo community in Assam.
 Gatka – It is a traditional combat training in which wooden sticks are used to simulate swords
in Punjab.
Hence, option (d) is correct.

Q 49.D
o Doklam is an area with a plateau and a valley, lying between Tibet's Chumbi Valley to the north,
Bhutan's Ha Valley to the east and India's Sikkim state to the west. It has been depicted as part of
Bhutan in the Bhutanese maps since 1961, but it is also claimed by China. To date, the dispute has not
been resolved despite several rounds of border negotiations between Bhutan and China.In June 2017
a military standoff occurred between China and India as China attempted to extend a road on the Doklam
plateau southwards near the Doka La pass and Indian troops moved in to prevent the Chinese. India
claimed to have acted on behalf of Bhutan, with which it has a 'special relationship'.

Q 50.B C.P PHOTOSTAT


The Gulf of Thailand, formerly the Gulf of Siam, is an inlet of the South China Sea.
9555944010
Bordered by Thailand, Cambodia, and the south-western edge of Vietnam, the gulf's maximum width
is 350 miles (560 km), and it extends approximately 450 miles (725 km) in length.
The Gulf of Thailand is quite shallow along the coastal areas, and those waters provide lucrative fishing
grounds. Many rivers flow into the Gulf of Thailand, with the Chao Phraya being the most significant.

16 www.visionias.in ©Vision IAS

https://www.facebook.com/pdf4exams/
Downloaded From:- https://telegram.me/pdf4exams https://telegram.me/upsc_zone

Q 51.A
The Mediterranean type of climate is characterized by very distinctive climatic features. The summers
have a relatively warm and dry summer is experienced with off-shore trade winds. A temperature of
roughly 76˚F is observed in Rome. Rainfall is concentrated in the winters with on-shore westerlies
bringing cyclonic rain. Hence, statement 1 is correctly matched.
Sydney experiences China type of climate, also known as the Warm Temperate Eastern Margin type
which is characterized by a warm moist summer and a cool dry winter. Hence, statement 2 is not
correctly matched.
In Kuala Lumpur the hot and wet equatorial type of climate is observed. There is uniformity of
temperature throughout the year. Cloudiness and precipitation help to moderate the daily
temperature. Hence, statement 3 is correctly matched.

Q 52.A
Minor Relief Features :
Some minor but significant features predominate in different parts of the oceans are:
Mid-Oceanic Ridges : A mid-oceanic ridge is composed of two chains of mountains separated by a large
depression. The mountain ranges can have peaks as high as 2,500 m and some even reach above the
ocean’s surface. Iceland, a part of the mid Atlantic Ridge, is an example.
Seamount : It is a mountain with pointed summits, rising from the seafloor that does not reach the

T
surface of the ocean. Seamounts are volcanic in origin. These can be 3,000-4,500 m tall. The Emperor

A
seamount, an extension of the Hawaiian Islands in the Pacific Ocean, is a good example.
Submarine Canyons : These are deep valleys, some comparable to the Grand Canyon of the Colorado
10 ST
river. They are sometimes found cutting across the continental shelves and slopes, often extending from
the mouths of large rivers. The Hudson Canyon is the best known submarine canyon in the world.
40 TO
Guyots : It is a flat topped seamount. They show evidences of gradual subsidence through stages to
become flat topped submerged mountains. It is estimated that more than 10,000 seamounts and guyots
C.P PHOTOSTAT
exist in the Pacific Ocean alone.
Atoll : These are low islands found in the tropical oceans consisting of coral reefs surrounding a central
9555944010
94 O

depression. It may be a part of the sea (lagoon), or sometimes form enclosing a body of fresh, brackish, or
55 PH

highly saline water.

Q 53.B
Only pair 3 is correctly matched.
95 .P

 Asteroids are generally larger chunks of rock that come from the asteroid belt located between
the orbits of Mars and Jupiter. Sometimes their orbits get perturbed or altered and some asteroids
C

end up coming closer to the Sun, and therefore closer to Earth.Comets are much like asteroids,
but might have a more ice, methane, ammonia, and other compounds that develop a fuzzy, cloud-
like shell called a coma – as well as a tail — when it gets closer to the Sun. Space debris smaller
than an asteroid are called meteoroids. A meteoroid is a piece of interplanetary matter that is
smaller than a kilometer and frequently only millimeters in size. Most meteoroids that enter the
Earth’s atmosphere are so small that they vaporize completely and never reach the planet’s
surface.
 If any part of a meteoroid survives the fall through the atmosphere and lands on Earth, it is called
meteorite. Meteor is the flash of light that we see in the night sky when a small chunk of
interplanetary debris burns up as it passes through our atmosphere.

Q 54.D
All the statements are correct.
Government of India is introducing ―Intensified Mission Indradhanush (IMI)‖ in select districts and urban
areas of the country to achieve the target of more than 90% coverage.
The strategy of IMI is to cover all left outs and drop outs in select districts and urban areas with low
routine immunization coverage in a specific time-frame (December 2018). These districts will focus
on improving immunization coverage through need based interventions in Intensified Mission
Indradhanush drives, based on a comprehensive gap analysis, with strengthened involvement of relevant

17 www.visionias.in ©Vision IAS

https://www.facebook.com/pdf4exams/
Downloaded From:- https://telegram.me/pdf4exams https://telegram.me/upsc_zone

non-health departments and enhanced accountability frameworks. The gains thus achieved need to be
sustained through strengthening health systems and microplanning by incorporating IMI sessions into
routine immunizations sessions.
The key to effective implementation of targeted rapid interventions to improve the routine immunization
coverage are:
 Inter-ministerial and inter-departmental coordination
 Action based review mechanism
 Intensive monitoring and accountability framework

IMI will focus on children up to 2 years of age and pregnant women who have missed out on routine
immunization. However, vaccination on demand to children up to 5 years of age will be provided
during IMI rounds. The sustainability of IMI will be assessed through a survey, and the decision to
conduct another phase of IMI will be based on the findings of this survey.

Q 55.C
 Statement 1 is correct: India- Myanmar boundary runs roughly along the watershed between the
Brahmaputra and Ayeyarwady [Irrawaddy]. It passes through thickly forested regions, with Mizo
Hills, Manipur and Nagaland on the Indian side and Chin Hills, Naga Hills and Kachin state on the
Myanmar side.
 Statement 2 is not correct:India shares longest international border with Bangladesh. It is 4,096 km
long. This boundary has been determined under the Radcliffe Award which divided the erstwhile province
of Bengal into two parts.

C.P PHOTOSTAT
9555944010

 Statement 3 is correct: India shares boundary with Afghanistan through Wakhan corridor. The
Wakhan Corridor is about 210km long (130 miles) long.

Q 56.D
The midday sun is exactly overhead at least once a year on all latitudes in between the Tropic of Cancer
and the Tropic of Capricorn. The midday sun never shines overhead on any latitude beyond the Tropic of
Cancer and the Tropic of Capricorn. Hence, statement 1 is not correct.
The angle of the sun’s rays goes in decreasing, and not increasing, towards the poles. Hence, statement 2
is not correct.

18 www.visionias.in ©Vision IAS

https://www.facebook.com/pdf4exams/
Downloaded From:- https://telegram.me/pdf4exams https://telegram.me/upsc_zone

Q 57.C
Elements in earth’s crust by Weight(%) Elements in earth as a whole by weight (%)
1. Oxygen 46.60 1. Iron 32.1
2. Silicon 27.72 2. Oxygen 30
3. Aluminium 8.13 3. Silicon 15
4. Iron 5.00 4. Magnesium 13
5. Calcium 3.63 5. Sulphur 2.9
6. Sodium 2.83 6. Nickel 1.8
7. Potassium 2.59 7. Calcium 1.5
8. Magnesium 2.09 8. Aluminium 1.4
9. Others 1.41
o In terms of its constituent elements, the mantle is made up of 44.8% oxygen, 21.5% silicon, and
22.8% magnesium. There’s also iron, aluminum, calcium, sodium, and potassium. These elements are
all bound together in the form of silicate rocks, all of which take the form of oxides. The most
common is Silicon dioxide (SiO2) at 48%, followed by Magnesium Oxide (MgO) at 37.8%. Examples
of rocks that you might find inside the mantle include: olivine, pyroxenes, spinel, and garnet.
o Hence, option (c) is correct.

Q 58.D

T
o Indian Coast Guard Ship Shaurya is an offshore patrol vessel designed and built indigenously by Goa

A
Shipyard Ltd (GSL). It will be deployed extensively for Exclusive Economic Zone surveillance, apart
from safeguarding India’s maritime interests. The ship has capacity to carry twin engine light helicopter
10 ST
and five high speed boats including quick response interceptor boats for quick boarding operations, search
and rescue, law enforcement and maritime patrol. It is fitted with state-of-the-art navigation and
40 TO
communication equipment, sensors and machineries. It is also capable of carrying pollution response
equipment to combat oil spill contamination. Hence, option (d) is correct.
C.P PHOTOSTAT
Q 59.C
9555944010
94 O

Normally, temperature decreases with increase in elevation. It is called normal lapse rate. At times, the
55 PH

situations is reversed and the normal lapse rate is inverted. It is called inversion of temperature. A long
winter night with clear skies and still air is ideal situation for inversion. Smoke and dust particles get
collected beneath the inversion layer and spread horizontally to fill the lower strata of the atmosphere.
Dense fogs in mornings are common occurrences especially during winter season. This
95 .P

inversion commonly lasts for few hours until the sun comes up and beings to warm the earth. Hence, both
statements 1 and 2 are correct.
C

Q 60.C
El Nino Modoki is a coupled ocean-atmosphere phenomenon in the tropical Pacific. It is different from
another coupled phenomenon in the tropical Pacific namely, El Nino. Conventional El Nino is
characterized by strong anomalous warming in the eastern equatorial Pacific. Whereas, El Nino Modoki is
associated with strong anomalous warming in the central tropical Pacific and cooling in the eastern
and western tropical Pacific. Hence, statement 1 is correct.
The El Niño Modoki events impact the formation of tropical cyclones over north Indian Ocean.The reason
why El Nino Modoki brings only fewer number of cyclones in the Bay of Bengal is because of descending
air conditions. The descending causes dry conditions not conducive for cyclone formation. Also, an El
Nino Modoki creates stronger divergence over the western Pacific and Bay of Bengal compared to El
Nino. Divergence (opposite of convergence) means surface winds move away from each other and result
in low relative vorticity (rotational flow of winds). These conditions are not conducive for cyclones. This
explains why Bay of Bengal region (close to western Pacific) has fewer cyclones during an El Nino
Modoki.
On the other hand, there is large convergence over the Arabian Sea during an El Nino Modoki
explaining the large number of cyclones in that region. A statistical analysis of the El Nino and El
Nino Modoki years between 1979-2004 was conducted. It was found that there were four El Nino years
and seven El Nino Modoki years during this period. The number of cyclones per year show significant

19 www.visionias.in ©Vision IAS

https://www.facebook.com/pdf4exams/
Downloaded From:- https://telegram.me/pdf4exams https://telegram.me/upsc_zone

differences indicating that El Nino Modoki years are conducive for cyclone formation over Arabian Sea
while El Nino is conducive for cyclones over the Bay of Bengal. Hence, statement 2 is correct.

Q 61.A
Option (a) is the correct answer.
Gran Chaco: In the lowlands of northern Argentina and western Paraguay, it is dry during winter and wet
during summer. Rains are heavy.These are warm temperate forests. This region is covered with thick
forests and grasslands known as the Gran Chaco.
Option (b) : On either side of the Amazon lies the belt which has the Savanna type of climate. This is the
region of the grasslands. In the north, in the Orinoco river basin, they are locally known as the LLANOS.
In the south, in central Brazil, they are known as CAMPOS. In this region, there is a distinct dry period
and the rain occurs mainly in summer.
Option (c) : Mangroves are a group of trees and shrubs that live in the coastal intertidal zone. All of
these trees grow in areas with low-oxygen soil, where slow-moving waters allow fine sediments to
accumulate. Mangrove forests only grow at tropical and subtropical latitudes near the equator because
they cannot withstand freezing temperatures.
Option (d) : The Amazon Basin which lies close to the equator has an equatorial type of climate. It is
hot and wet all the year round. This region is therefore covered with equatorial rain- forests locally known
as the SELVAS.

Q 62.A
National Pharmaceutical Pricing Authority [NPPA] is an independent body under Department of
Pharmaceuticals under Ministry of Chemicals and Fertilizers. Its functions are to fix/revise the controlled
bulk drugs prices and formulations.
National List of Essential Medicines contains 376 medicines. Criteria for inclusion into this list includes
public health emergency, cost effective medicine etc. Once a drug or medical device is included in
C.P PHOTOSTAT
NLEM, its price can be capped by NPPA. Recently, NPPA capped the prices of knee implants.

9555944010
Hence, options 1 and 2 are correct.
Surgical procedures are not covered under this. Hence, option 3 is not correct.

Q 63.D
The Pacific Ocean is the largest ocean having area larger than the combined area of all continents. It is
also deeper than any other ocean
In order of size the major oceans are – the Pacific Ocean, the Atlantic Ocean, the Indian Ocean and
the Arctic Ocean. The extension of the Pacific, the Atlantic and the Indian Oceans around Antarctica is
called the Southern Ocean.

Q 64.C
All the options are correct.
The factors influencing the climate of Europe are:
1. Westerlies
2. North Atlantic Drift
3. Alpine System
The Gulf Stream is the most important ocean-current system in the northern hemisphere, which stretches
from Florida to north-western Europe. It incorporates several currents: the Florida current, the Gulf
Stream itself, and an eastern extension, the North Atlantic Drift. The relatively warm waters of the North
Atlantic Drift are responsible for moderating the climate of western Europe, so that winters are less cold
than would otherwise be expected at its latitude. Without the warm North Atlantic Drift, the UK and other
places in Europe would be as cold as Canada, at the same latitude. For example, without this steady
stream of warmth the British Isles winters are estimated to be more than 5 °C cooler, bringing the average
December temperature in London to about 2°C.
Europe lies in the region of the Westerlies. As such, most of the time the wind blows from the south-west.
Since there is no mountain from north to the south, there is no obstruction for these winds blow deeper
into the land and modify the temperature. The warm waters of the North Atlantic Drift keep the seas along

20 www.visionias.in ©Vision IAS

https://www.facebook.com/pdf4exams/
Downloaded From:- https://telegram.me/pdf4exams https://telegram.me/upsc_zone

western Europe ice-free. The Westerlies blowing over these currents carry warmth further inland. On their
way, they also pick up moisture and cause a fair amount of rainfall. Since there are permanent winds,
rainfall is also fairly well distributed all through this year. It is generally heavy in the west and decreases
towards east.
The moderating influence of the Westerlies and nearness to the seas and oceans keep summers warm and
winters cool in western Europe. The temperature equable and rainfall is well distributed throughout the
year. In winter, fogs are common. This type of climate is typically maritime and is known as Western
Europan type.
The location of the Alps, as well as the great variations in their elevations and exposure, give rise to
extreme differences in climate, not only among separate ranges but also within a particular range itself.
Temperature extremes and annual precipitation are related to the physiography of the Alps. The valley
bottoms clearly stand out because generally they are warmer and drier than the surrounding heights. In
winter nearly all precipitation above 5,000 feet is in the form of snow, and depths from 10 to 33 feet or
more are common. Snow cover lasts from approximately mid-November to the end of May at the 6,600-
foot level, blocking the high mountain passes; nevertheless, relatively snowless winters can occur. Mean
January temperatures on the valley floors range from 23° to 39° F (−5° to 4° C) to as high as 46° F (8° C)
in the mountains bordering the Mediterranean, whereas mean July temperatures range between 59° and
75° F (15° and 24° C). Temperature inversions are frequent, especially during autumn and winter, and the

T
valleys often fill with fog and stagnant air for days at a time. At those times the levels above 3,300 feet

A
can be warmer and sunnier than the low-lying valley bottoms. Winds can play a prominent role in daily
10 ST
weather and microclimatic conditions.

Q 65.B
40 TO

It is the date when humanity annual demand on nature exceeds what Earth can regenerate over the
C.P PHOTOSTAT
entire year. It is calculated by WWF and Global Footprint Network.
It signifies that we have emitted more carbon than the oceans and forests can absorb in a year, we caught
9555944010
94 O

more fish, felled more trees, harvested more, and consumed more water than the Earth was able to
55 PH

produce in the same period.


In 2017, Earth Overshoot Day fell on August 2, the earliest date since ecological overshoot began in the
early 1970s.
Q 66.A
95 .P

o Biological activity is an integral part of formation of soil, as it helps in adding organic matter, moisture
retention, nitrogen etc. But there is a difference between soils of cold and warm climate due to intensity of
C

bacterial activity. In humid, tropical and equatorial climate, there is high precipitation because of
which soil leaching is high which makes the soils low in humus content . Also, bacterial growth and
action is intense and dead vegetation is rapidly oxidized leaving very low humus content in the soil. On
the other hand, humus accumulates in cold climate and bacterial growth is also low. Because of low
bacterial growth, organic matter remains undecomposed and layers of peat develop in sub-arctic
and tundra climate. Hence, soils of cold climate have high humus content.
o Hence, only statement 1 is correct.

Q 67.A
Erosional features:
By wind- wind eroded basins
By glaciers- U-shaped valleys
By sea waves- arches and sea stacks
Depositional features:
By wind- sand dunes
By glaciers- moraines
By sea waves- beaches, sand bars

Q 68.B
The correct order is Krishna>Brahmaputra>Mahanadi>Narmada
21 www.visionias.in ©Vision IAS

https://www.facebook.com/pdf4exams/
Downloaded From:- https://telegram.me/pdf4exams https://telegram.me/upsc_zone

Krishna Basin has an area of about 2,58,948sq km. River Krishna originates from Satara district of
Maharashtra and has a length of 1400 km. It flows through Maharashtra, Karnataka, Telangana and
Andhra Pradesh emptying into the Bay of Bengal. Its principal tributaries are Ghatprabha, Malprabha,
Bhima, Musi, Munner.
Brahmaputra Basin has an area about 1,94,413sq km with a total length of about 2900 km of which it
travels only 916 km in India. Its principal tributaries are Lohit, Dibang, Subansiri, Kameng, Teesta,
Dikhow etc.
Mahanadi Basin has roughly an area of about 1,41,589sq km which soreads over five states of
Chattisgarh, Odisha, Madhya Pradesh, Jharkhand, and Maharashtra. Its principal tributaries are Seonath,
Hasdeo, Ong, Ib, Tel and Jonk.
Narmada basin extends over states of Madhya Pradesh, Gujarat, Maharashtra and Chhattisgarh having an
area of 98,796 Sq.km which is nearly 3% of the total geographical area of the country. It is bounded by
the Vindhyas on the north, by the Maikala range on the east, by the Satpuras on the south and by the
Arabian Sea on the west.

Q 69.B
The insolation received at the surface varies from about 320 Watt/m2 in the tropics to about 70 Watt/m2
in the poles. Maximum insolation is received over the subtropical deserts, where the cloudiness is the
least. Equator receives comparatively less insolation than the tropics. Generally, at the same latitude the
insolation is more over the continent than over the oceans. In winter, the middle and higher
latitudes receive less radiation than in summer.

Q 70.C
o Statement 1 is correct: Earthquake waves are basically of two types — body waves and surface waves.
Body waves are generated due to the release of energy at the focus and move in all directions travelling
C.P PHOTOSTAT
through the body of the earth. Hence, the name body waves. The body waves interact with the surface
rocks and generate new set of waves called surface waves. These waves move along the surface on the

o
9555944010
interface of earth and atmosphere.
The velocity of waves changes as they travel through materials with different densities. The denser the
material, the higher is the velocity. Their direction also changes as they reflect or refract when coming
across materials with different densities.
o Statement 2 is not correct: Surface waves do not travel in interior of earth like body waves. This
characteristic of the S-waves is quite important. It has helped scientists to understand the structure of the
interior of the earth. Reflection causes waves to rebound whereas refraction makes waves move in
different directions. The variations in the direction of waves are inferred with the help of their record on
seismograph.
o Statement 3 is correct: The surface waves are the last to report on seismograph. These waves are more
destructive. They cause displacement of rocks, and hence, the collapse of structures occurs.

Q 71.D
The transformation of water vapour into water is called condensation.
In free air, condensation results from cooling around very small particles termed as hygroscopic
condensation nuclei. (Free air is air not under restraint by pressure or flow or any kind of turbulence.)
Particles of dust, smoke and salt from the ocean are particularly good nuclei because they absorb water.
The most favourable condition for condensation is the decrease in air temperature.
Condensation is influenced by the volume of air, temperature, pressure and humidity.
After condensation the water vapour or the moisture in the atmosphere takes one of the following forms
— dew, frost, fog and clouds.

Q 72.B
The Union cabinet has recently approved proposal to change the electoral laws to allow NRIs to vote in
the LokSabha and assembly elections through proxy. Earlier, this was permitted only to service personnel.
22 www.visionias.in ©Vision IAS

https://www.facebook.com/pdf4exams/
Downloaded From:- https://telegram.me/pdf4exams https://telegram.me/upsc_zone

NRIs electors will have to appoint a nominee afresh for each election — one person can act as proxy for
only one overseas voter.
Service voters can cast their vote through post as well but this is not permitted for NRIs.
Overseas citizen of India have no voting rights in Indian election process.
Q 73.B

A T
10 ST
40 TO

C.P PHOTOSTAT
9555944010
94 O
55 PH

Q 74.B
95 .P

The earth as a whole does not accumulate or loose heat. It maintains its temperature. This can happen only
if the amount of heat received in the form of insolation equals the amount lost by the earth
C

through terrestrial radiation.


Consider that the insolation received at the top of the atmosphere is 100 per cent. While passing through
the atmosphere some amount of energy is reflected, scattered and absorbed. Only the remaining part
reaches the earth surface. The remaining 65 units are absorbed, 14 units within the atmosphere and 51
units by the earth’s surface. Hence statement 1 is not correct.
Roughly 35 units are reflected back to space even before reaching the earth’s surface. Of these, 27
units are reflected back from the top of the clouds and 2 units from the snow and ice-covered areas
of the earth (cryosphere). The reflected amount of radiation is called the albedo of the earth. Hence
statement 2 is correct.

Q 75.C
o Lt General D.B. Shekatkar panel recently gave recommendations for enhancing combat capability and
rebalancing defence expenditure. The panel was constituted in 2016 by the Central government. Some of
its recommendations are:
1. Judicious use of resources by optimization of supply, transport and ordnance infrastructure and the
closure of 39 military farms and several military postal departments in so-called peace locations.
2. Improve operational efficiency of the Army: Redeployment and Restructuring of approximately
57,000 posts of officers, soldiers and civilians in the Army.
3. The panel recommended raise in the retirement age of jawans by two years, that will save the army
significant amount on pensions and training.

23 www.visionias.in ©Vision IAS

https://www.facebook.com/pdf4exams/
Downloaded From:- https://telegram.me/pdf4exams https://telegram.me/upsc_zone

Q 76.A
o Statement 1 is correct: This is the most widely spread and important soil. In fact, the entire northern
plains are made of alluvial soil. These have been deposited by three important Himalayan river systems–
the Indus, the Ganga and the Brahmaputra. These soils also extend in Rajasthan and Gujarat through a
narrow corridor. Alluvial soil is also found in the eastern coastal plains particularly in the deltas of the
Mahanadi, the Godavari, the Krishna and the Kaveri rivers.
o Statement 2 is correct: Alluvial soils are generally rich in potash and lime but poor in phosphorous.
o Statement 3 is not correct: Alluvial soils are fertile because of having fine silt particles deposited
annually by floods. These soils are poor in humus and nitrogen except the alluvial of Ganga deltaic
region. Hence, it also support large variety of rabi and kharif crops.

Q 77.C
JananiSurakshaYojana (JSY) and JananiShishuSurakshaKaryakaram (JSSK), the number of institutional
deliveries have improved. These programmes entitles all pregnant women delivering in public health
institutions to absolutely free ante-natal checkups, delivery including Caesarean section, post-natal care
and treatment of sick infants till one year of age. Hence, option 1 is correct.
RashtriyaBalSwasthyaKaryakram (RBSK), an introduction of child health screening for 4Ds i.e. defects at
birth, deficiencies, diseases, development delays and their management among the children 0-18 years of
age. Hence, option 3 is correct.
The Mid Day Meal is the world’s largest school feeding programme reaching out to about 12 crore
children in over 12.65 lakh schools/EGS centres across the country. Mid day Meal scheme is serving
primary and upper primary school children in entire country. Hence, option 2 is not correct.

Q 78.D C.P PHOTOSTAT


Carbon Dioxide is meterologically a very important gas as it is transparent to the incoming solar radiation
but opaque to 9555944010
the outgoing terrestrial radiation. It absorbs the terrestrial radation and reflects some part of
it back towards the Earth's surface. Thus, it is also largely responsible for the greenhouse effect. Carbon
Dioxide (0.036) is the fourth largest component of the Earth's atmosphere after Nitrogen (78.8),
Oxygen (20.95) and Argon (0.93). Hence, statement 1 is not correct.
The atmosphere is composed of gases, water vapour and dust particles. The proportion of gases changes in
the higher layers of the atmosphere in such a way that carbon dioxide and water vapourare found only up
to 90 km from the surface of the Earth. The troposphere extends upward to about 10 km above sea
level. Hence, statement 2 is not correct.

Q 79.A
In the northern hemisphere the land surface area is much larger than in the southern hemisphere. Hence,
the effects of land mass and the ocean currents are well pronounced. In January the isotherms deviate to
the north over the ocean and to the south over the continent. Hence, statement 1 is correct.
The effect of the ocean is well pronounced in the southern hemisphere. Here the isotherms are more or
less parallel to the latitudes and the variation in temperature is more gradual than in the northern
hemisphere. The isotherm of 20° C, 10° C, and 0° C runs parallel to 35° S, 45° S and 60° S latitudes
respectively. Hence, statement 2 is correct.
No two isotherms ever intersect each other as it would mean that a particular place has two different
temperatures at the same time which is impossible. Hence, statement 3 is not correct.

Q 80.B
o Ghantasala was a renowned Buddhist centre. Recently, Andhra Pradesh government has passed an
order to build a 70 ft. Buddha statue in Ghantasala in Krishna district in Andhra Pradesh. The new
facility will be themed on the Mahaparinirvana of the Buddha. The word 'Mahaparinirvana' refers to the

24 www.visionias.in ©Vision IAS

https://www.facebook.com/pdf4exams/
Downloaded From:- https://telegram.me/pdf4exams https://telegram.me/upsc_zone

ultimate state of Nirvana (everlasting, highest peace and happiness) entered by an Awakened Being
(Buddha) or 'arhat' at the moment of physical death; but it can also refer to that same state reached during
such a being's physical lifetime too.
o Hence, option (b) is correct.

Q 81.A
Government has accorded its approval for creation of a non-lapsable pool in the Public Account for
secondary and higher, education known as 'Madhyamik and UchchtarShikshaKosh' (MUSK) into which
all proceeds of 'Secondary and Higher Education Cess' will be credited.
The funds arising from the MUSK would be utilized for schemes in the education sector which
would be available for the benefit of students of secondary and higher education, all over the
country.
In connection with the above fund, the Union Cabinet also accorded its approval to the following:
(i) Administration and maintenance of the above pool by Ministry of Human Resource Development.
(ii) Accruals from the Cess would be utilized in the ongoing schemes of Secondary and Higher Educatin.
However, the Ministry of Human Resources Development can allocate funds for any future
programme/scheme of secondary and higher education, based on the requirement, as per prescribed
procedure,

T
MUSK would be maintained as a Reserve Fund in the non-interest bearing section of the Public Accounts

A
of India. The major benefit will be enhancing access to secondary and higher education through
10 ST
availability of adequate resources, while ensuring that the amount does not lapse at the end of financial
year.
40 TO

Q 82.B
C.P PHOTOSTAT
A Chinook wind is a warm, dry wind originating from the Pacific Ocean that blows eastward over the
Rocky Mountains, then down their slopes onto the prairies below. Hence, statement 1 is not correct.
9555944010
94 O

Mistral, Italian maestrale, cold and dry strong wind in southern France that blows down from the north
along the lower Rhone River valley toward the Mediterranean Sea. It may blow continuously for
55 PH

several days at a time, with velocities that average about 74 km (about 45 miles) per hour, and reach to a
height of 2 to 3 km (about 1.2 to 1.9 miles). It is strongest and most frequent in winter, and it sometimes
causes considerable damage to crops. The velocity of the wind is intensified as it blows down from the
highlands to the coast and by the ―jet effect‖ that results as it is funneled through the narrow Rhône valley.
95 .P

As the winds move out over the Rhône delta, they can reach velocities of 130 km (about 80 miles) per
hour. Hence, statement 2 is correct.
C

Sirocco, warm, humid wind occurring over the northern Mediterranean Sea and southern Europe,
where it blows from the south or southeast and brings uncomfortably humid air. The sirocco is
produced on the east sides of low-pressure centres that travel eastward over the southern Mediterranean.
It originates over North Africa as a dry wind and picks up moisture as it crosses the
Mediterranean. Hence, statement 3 is correct.

Q 83.A
The eastern coast of India is the most cyclone affected region. The cyclone prone states are: West Bengal,
Orissa, Andhra Pradesh and Tamil Nadu. Western coast is affected by the cyclones which originate in the
Arabian Sea. Gujarat on the west coast is most affected by cyclones. The coastal areas and interior of
Maharashtra are affected by cyclones too. More cyclones originate in the Bay of Bengal and the Arabian
Sea than any other seas of the world. Hence, statement 1 is correct.
The devastating effect of cyclone goes on decreasing as it moves away from sea. This is because the
moisture supply has been cut. Coastal cities are more prone to tropical cyclone. Vishakhapatnam is a
coastal city. Hence, statement 2 is not correct.

Q 84.C
 LTIF was created in 2016 with an initial corpus of Rs. 20,000 crore for funding and fast tracking the
implementation of incomplete major and medium irrigation projects. Hence statement 1 is correct.
25 www.visionias.in ©Vision IAS

https://www.facebook.com/pdf4exams/
Downloaded From:- https://telegram.me/pdf4exams https://telegram.me/upsc_zone

 It aims to bridge the resource gap and facilitate completion of these projects during 2016-2020. 23
projects (priority-I) have been identified to be completed by 2016-17, 31 projects (priority –II) have been
identified to be completed by 2017-18 and balance 45 projects (priority – III) have been identified to be
completed by 2019-20.
 LTIF has been instituted in NABARD under PradhanMantriKrishiSinchayeeYojana. Hence, statement 2
is correct.

Q 85.B
Option (b) is the correct answer.

C.P PHOTOSTAT
9555944010

 A large part of Australia is a rain-thirsty land. Only 4% of its land is under cultivation. But people
have managed their limited land and water resources very well. It is known for its pastoral industries
which are pursued on modern and scientific lines.
 Artesian well, deep drilled well through which water is forced upward under pressure. The water in an
artesian well flows from an aquifer, which is a layer of very porous rock or sediment, usually
sandstone, capable of holding and transmitting large quantities of water. The geologic conditions
necessary for an artesian well are an inclined aquifer sandwiched between impervious rock layers
above and below that trap water in it. Water enters the exposed edge of the aquifer at a high elevation
and percolates downward through interconnected pore spaces. The water held in these spaces is under
pressure because of the weight of water in the portion of the aquifer above it. If a well is drilled from
the land surface through the overlying impervious layer into the aquifer, this pressure will cause the
water to rise in the well. The largest artesian system in the world underlies nearly all of East and
South Australia.

Q 86.B
The East African Rift (EAR) is an active continental rift resulting from the splitting of the African plate
into two. The EAR has two main branches: the Eastern Rift Valley, which comprises the Main Ethiopia

26 www.visionias.in ©Vision IAS

https://www.facebook.com/pdf4exams/
Downloaded From:- https://telegram.me/pdf4exams https://telegram.me/upsc_zone

Rift running from the Afar Triple Junction and continues south as the Kenyan Rift Valley, and the
Western Rift Valley.
The following map shows the countries traversed by the EAR.

A T
10 ST
40 TO

C.P PHOTOSTAT
9555944010
94 O
55 PH

Q 87.B
Direct Sources of information about the interior:
95 .P

1. Deep Ocean Drilling Project


2. Integrated Ocean Drilling Project
C

3. Volcanic eruption
Hence, options 1 and 2 are correct.
Indirect Sources of information about the interior
1. The meteors that reach the earth
2. gravitation,
3. magnetic field, and
4. seismic activity.
Hence, options 3 and 4 are incorrect.

Q 88.A
A Bird's foot delta form where sea waves are weak in comparison to the strength of the river. The
deposition of fine sediments extends out beyond the coastline. The channels of the river divide up into
distributaries and looks like bird's feet. An example is the Mississippi Delta. Hence, pair 1 is correctly
matched.
An Arcuate delta is in the shape of an arc. It looks like an upside down triangle when viewed from above.
Arcuate deltas form where coarser materials such as gravel are deposited. The river deposits the material
and new river channels are formed. The river Nile is an example. Hence, pair 2 is correctly matched.
Lacustrine deltas form at the mouth of a river when it flows into a Lake. River Narmada forms an Esturine
delta as it flows directly into the Arabian Sea. Hence, pair 3 is not correctly matched.

27 www.visionias.in ©Vision IAS

https://www.facebook.com/pdf4exams/
Downloaded From:- https://telegram.me/pdf4exams https://telegram.me/upsc_zone

Q 89.B
There are three stages in the evolution of the present atmosphere. The first stage is marked by the loss of
primordial atmosphere. In the second stage, the hot interior of the earth contributed to the evolution of the
atmosphere. Finally, the composition of the atmosphere was modified by the living world through the
process of photosynthesis. Oceans began to have the contribution of oxygen through the process of
photosynthesis. Eventually, oceans were saturated with oxygen, and 2,000 million years ago, oxygen
began to flood the atmosphere. Hence, option b is correct.

Q 90.C
o Winds and water are the powerful agents of soil erosion because of their ability to remove soil and
transport it. Water erosion occurs extensively in different parts of India, takes place mainly in the form of
sheet and gully erosion.
o In regions with heavy rainfall and steep slopes, erosion by running water is significant. Gully erosion is
common on steep slopes by the heavy rainfall. A region with large number of deep gullies or ravines
is called a badland topography. On the other hand, erosion on the flat lands after a heavy shower is
known as Sheet erosion. However, wind erosion is significant in arid and semi-arid regions.
o Hence, option (c) is correct.

Q 91.B
The atmosphere consists of different layers with varying density and temperature. Density is highest near
the surface of the earth an decreases with increasing altitude. The column of atmosphere is divided into
five differen layers depending upon the temperature condition. They are: troposphere,
stratosphere mesosphere, thermosphere and exosphere. The troposphere is the lowermost layer of the
atmosphere. Its average height is 13 km and extends roughly to a height of 8 km near the poles and about
18 km at the equator. Thickness of the troposphere is greatest at the equator because heat is transported to
C.P PHOTOSTAT
great heights by strong convectional currents. This layer contains dust particles and water vapour. All

9555944010
changes in climate and weather take place in this layer. The temperature in this layer decreases at the rate
of 1 °C for every 165m of height. This is the most important layer for a biological activity.
The zone separating the tropsophere from stratosphere is known as the tropopause. The air temperature at
the tropopause is about minus 800C over the equator and about minus 45oC over the poles. The
temperature here is nearly constant, and hence, it is called the tropopause.
The stratosphere is found above the tropopause and extends up to a height of 50 km. One important
feature of the stratosphere is that it contains the ozone layer. This layer absorbs ultra-violet radiation
and shields life on the earth from intense, harmful form of energy. The mesosphere lies above the
stratosphere, which extends up to a height of 80 km. In this layer, once again, temperature
starts decreasing with the increase in altitude and reaches up to minus 100°C at the height of 80 km. The
upper limit of mesosphere is known as the mesopause.
The thermosphere is a layer of Earth's atmosphere. The thermosphere is directly above the mesosphere
and below the exosphere. It extends from about 90 km (56 miles) to between 500 and 1,000 km (311 to
621 miles) above our planet. Temperatures climb sharply in the lower thermosphere (below 200 to 300
km altitude), then level off and hold fairly steady with increasing altitude above that height.
Hence, only statements 2 and 4 are correct.

Q 92.B
o Republic of Djibouti, is a country located in the Horn of Africa. It is bordered by Eritrea in the north,
Ethiopia in the west and south, and Somalia in the southeast. The remainder of the border is formed by the
Red Sea and the Gulf of Aden at the east.
o Recently China have developed its first military base outside its territory in Djibouti. The Chinese naval
base in Djibouti is a military base operated by People's Liberation Army Navy (PLAN) of the People's
Republic of China at Djibouti in the Horn of Africa. It is the first overseas military base of the PLAN, and
was constructed west of Djibouti City, adjacent to the Port of Doraleh.

28 www.visionias.in ©Vision IAS

https://www.facebook.com/pdf4exams/
Downloaded From:- https://telegram.me/pdf4exams https://telegram.me/upsc_zone

Q 93.B
o Weathering causes the disintegration of rock near the surface of the earth. Plant and animal life,
atmosphere and water are the major causes of weathering. Weathering breaks down and loosens the
surface minerals of rock so they can be transported away by agents of erosion such as water, wind and ice.
o Processes of weathering are:
 Physical weathering: Block disintegration, Granular disintegration, exfoliation, frost action
 Chemical weathering: solution, oxidation, carbonation, hydration
 Biological weathering: by animals and insect, vegetation, man, etc

Q 94.C
o The energy emanating from within the earth is the main force behind endogenic geomorphic
processes. This energy is mostly generated by radioactivity, rotational and tidal friction and primordial
heat from the origin of the earth. This energy due to geothermal gradients and heat flow from within
induces diastrophism and volcanism in the lithosphere. Due to variations in geothermal gradients
and heat flow from within, crustal thickness and strength, the action of endogenic forces are not
uniform and hence the tectonically controlled original crustal surface is uneven.
o Diastrophism and volcanism are endogenic geomorphic processes. All processes that move, elevate or
build up portions of the earth’s crust come under diastrophism. Volcanism includes the movement of
molten rock (magma) onto or toward the earth’s surface and also formation of many intrusive and

T
extrusive volcanic forms.

A
o Hence, both the statements are correct.
10 ST
Q 95.A
o Peaty soils are found in areas of high rainfall and high humidity, where there is a good vegetation
40 TO

growth. Thus, large quantity of dead organic matter accumulates in these areas and give us a rich humus
C.P PHOTOSTAT
and organic content to the soils. They contain 10-40% of organic matter. It is widely found in northern
part of Bihar, southern part of Uttarakhand, coastal areas of West Bengal, Odisha and Tamil Nadu
9555944010
94 O

and in Kottayam and Alappuzha district of Kerala where it is called 'kari'. The peaty soils are black,
damp and spongy. It is acidic in nature which slows down decomposition and leads to the soil having
55 PH

fewer nutrients.

Q 96.D
95 .P

India’s only live volcano in the Andaman and Nicobar Islands is showing such dramatic movements
lately. According to the researchers at Goa-based National Institute of Oceanography (NIO), Barren
C

Island volcano located in the Andaman Sea had started showing activity in the year 1991 after being
dormant for over 150 years has once again started spewing ash. The team of scientists have confirmed in
their observation that during the daytime only ash clouds were observed.
India has a list of six volcanoes including the only live one, Barren Islands volcano which also happens to
be the only confirmed active volcano in South Asia. The first recorded volcanic eruption here dates back
to 1787.

Volcano Name State


1 Barren Island Andaman Islands
2 Narcondam Andaman Islands
3 Baratang Andaman Islands
4 Deccan Traps Maharashtra
5 Dhinodhar Hills Gujarat
6 Dhosi Hill Haryana

Q 97.D
o Tropical desert forests are also known as tropical thorn forests occur in the areas which receive rainfall
less than 50 cm. These consist of a variety of grasses and shrubs. In these forests, plants remain leafless

29 www.visionias.in ©Vision IAS

https://www.facebook.com/pdf4exams/
Downloaded From:- https://telegram.me/pdf4exams https://telegram.me/upsc_zone

for most part of the year and give an expression of scrub vegetation. Tussocky grass grows upto a height
of 2m as the under growth in these forests.
o These forests found in semi-arid areas of south west Punjab, Haryana, Rajasthan, Gujarat, Madhya
Pradesh and Uttar Pradesh. Important species found in these forests are babool, ber, date palm, khair,
neem, khejri, palas, etc.
o Hence, option (d) is correct.

Q 98.A
The orbital plane of the Earth is called the ecliptic, or the plane of the ecliptic. The angle between the
plane of the ecliptic and the Earth's equator is 23.5° Due to tilt of the earth's axis when northern
hemisphere is tilted towards the Sun it is summer in northern hemisphere (winter in southern hemisphere).
Due to tilt of the earth's axis when northern hemisphere is tilted away from the Sun it is winter in northern
hemisphere (summer in southern hemisphere). Tilt of the earth's axis also accounts for longer nights in
winter and the longer days in summer. Variation in length of day and night at different times of the
year is due to Revolution of the earth and its inclination to the plane of the ecliptic.

Q 99.B
The Kuroshio Current is a strong warm current in the Western North Pacific Ocean. Hence,
statement 1 is not correct.
It begins in the east coast of Taiwan and flows northeastward past Japan, where it merges with the easterly
drift of the North Pacific Current.
It is sometimes referred to as the Black Stream (English translation of Kurashio) and as the Japan Current.
It sustains the coral reefs of Japan and branches into the Sea of Japan as the Tsushima
Current. Hence, statement 2 is correct.

Q 100.B C.P PHOTOSTAT


Pairs 1 and 2 are correctly matched.
9555944010
 The Great Andaman group of islands in the north is separated by the Ten Degree Channel from the
Nicobar group in the south. The channel is 150 km wide.
 Palk strait separates the Tamil Nadu state of India and the Mannar district of northern province of Sri
Lanka.
 The two largest island groups of Lakshadweep Archipelago- Laccadives (to the north) and Minicoy
(to the south) are separated by the nine degree channel. Eight degree channel spearatesLakshdweep
islands and Maldives.

Copyright © by Vision IAS


All rights are reserved. No part of this document may be reproduced, stored in a retrieval system or transmitted
in any form or by any means, electronic, mechanical, photocopying, recording or otherwise, without prior
permission of Vision IAS.

30 www.visionias.in ©Vision IAS

https://www.facebook.com/pdf4exams/

You might also like